Sei sulla pagina 1di 69

FUNDAMENTALS OF MARINE VEHICLES

Dr B S Lee

PART 1 FUNDAMENTAL CONCEPTS

1.1. BASIC SHIP GEOMETRY

Most ships have one plane of symmetry, the centreplane, giving port-and-starboard
symmetry. Some vessels used in offshore industry may have two or more planes of
symmetry (e.g. semi-submersibles).

Since the length of most ships are bigger than breadth, it is natural that the direction
parallel to the length is referred to as longitudinal direction, as in, for example,
longitudinal stiffeners (or simply longitudinals). The athwartship direction is referred
to as transverse direction, as in transverse girders.

(a) Major Components of a Ship

A ship is essentially a box, usually water-tight, designed to float carrying


passengers and/or cargo and to propel itself. The main body of a ship is known as
the hull. The hull is enclosed on the top by the upper (or main) deck (that is, if it is
not an open ship). The hull will have the sideshell and the bottom (called
collectively a shell). The part of the ship above the hull is called the superstructure.
The spaces within the hull for carrying cargo are called holds, and the openings in
the deck to allow access to them are called hatches. Hatches usually have lips
around them to accommodate hatch covers, called hatch coaming. On ships
carrying liquid or gaseous cargo, the holds are called tanks.

(b) Principal Dimensions (see Figs 1.1 – 1.3)

Principal dimensions (sometimes called principal particulars) show the major


features of the vessel and include the following items:

Length between perpendiculars (LBP, LBP or LPP)


It is the distance between the aft perpendicular and the forward perpendicular.
This is often the length of the underwater body at the loaded draft, and therefore
is used in most calculations in naval architecture.

Length overall (LOA or LOA)


It is the distance from the extreme point at the forward end to a similar point at
the aft end. In most ships it exceeds LBP by a considerable amount. The excess
includes the overhang of the stern and stem. If there is a bulbous bow, LOA is
measured to the extreme point of the bulb.

Length on the waterline (LWL)


It is longitudinal distance of the waterline at any given draft. For most ships
LWL will be different for different draft.

Breadth moulded (B or BMLD)


This is the breadth of the ship at its broadest point, measured to the inside surface
of the side shell plating. This is the breadth commonly used for most
calculations, but is not the greatest breadth of the ship. The breadth extreme is

1
the breadth measured to the outside surface of the shell plating (equal to BMLD
plus twice the thickness of shell plating), or any side appendages if any (such as
fixed fenders). Breadth extreme will, therefore, indicate the clearance required
for the ship to pass through.

Depth moulded (D or DMLD)


It is the vertical distance measured amidships from the baseline (top of the keel
plating) to the underside of the deck plating at side. (Note that the depth at
centreline is usually greater than that at side – the difference is known as
camber).

Draft moulded (T)


This is the vertical distance from the moulded baseline to the waterline at which
the ship is floating. The extreme draft, of course, is the vertical distance from
underside of the keel plating to the waterline. The draft moulded is used for most
calculations, but draft extreme is important in determining the minimum depth of
water the ship can sail in (should give sufficient clearance because of a
phenomenon known as ‘squatting’).

Stern Superstructure Bow

Main Hull

Keel

Port quarter Port beam


Port bow

Centreline (centreplane)

Starboard bow
Starboard quarter Starboard beam

Fig. 1.1 Main parts of a ship


Fwd sheer
Aft sheer

AP FP

Summer load waterlineline

Midship Bulbous bow

Length between perpendiculars (LBP)

Length on waterline (LWL)

Length overall (LOA)

Fig. 1.2 Longitudinal principal dimensions

2
Breadth extreme

Fender
Depth mld (D) Camber

Breadth mld (B)


Baseline (top of keel)

Inside of side shell

Fig.1.3 Transverse principal dimensions

(c) Weights

When a ship is completed and ready for sea, but as yet empty other than just
enough fuel and lubricating oil and water to operate the engine, its mass is known
as lightweight. Whatever mass additional to the lightweight that the ship may have
on board at any time is called deadweight (DWT). The deadweight will include
pottable water, sludge, fuel and lube oil, stores, provision, crew and their gear and
ballast as well as the payload. At all times the sum of the lightweight and the
deadweight is equal to the (mass) displacement (∆).

The maximum deadweight that can be put on board is limited by the strength and
the seaworthiness of the ship, and is controlled by law (Load Line Regulations).
This limitation is shown clearly by markings of the maximum allowable draft on
the side of the ship (usually amidships) for a variety of different conditions. These
markings are known as ‘Plimsoll lines’.

(d) Other Features

There are some items which may be considered minor but sometimes are very
important for certain characteristics. These include the following:

Sheer
This is the progressive rise of the deck as one moves from midships towards the
ends. This feature is used to provide as much spare buoyancy as possible at the
narrow points of the ship, and also to protect the most vulnerable parts of the ship
to some extent. Small fishing boats often show high degree of sheer, and it is to
render the working platform at the side as near to water as possible.

In older ships the deck-at-side line was usually a parabola and the sheer is quoted
as its value at the forward and aft perpendiculars. The sheer forward was usually
twice as much as the sheer aft. In modern ships, however, the deck at side line
can either be flat with zero sheer over some distance on either side of amidships
and then rise as a straight line towards the ends, or be completely flat with no
sheer over the entire length.

3
Rise of floor
Some ships have a flat bottom (usually large ships), but others may have a bottom
rising from the centreline or a little off it. The height of the intersection of a line
drawn along this rising floor and the vertical line at the moulded breadth is called
rise of floor. High speed vessels often have high rise of floor (deep V-section) at
the bow region which becomes smaller (shallow V-section or U-section) towards
the stern.

Freeboard
This is the vertical distance from the waterline to the deck. Usually it will be a
minimum amidships and will increase towards the ends (due to sheer).
Obviously this is related to the maximum allowable load line, and is governed by
the Load Line Regulations.

1.2 FORM DEFINITIONS

(a) Lines Plan

Majority of ships has hull surfaces with three-dimensional curvature. This means
that the hulls cannot be represented by simple common geometric elements. Indeed
many techniques had to be devised to represent the hull surfaces in
mathematical/numerical forms so as to accommodate computerisation in the last
three decades. However, the traditional method of representing the hull form relies
on section shapes in planes parallel to the three major planes. The drawing
containing these sectional shapes is known as a lines plan and consists of

Half breadth plan (intersection of horizontal planes with ship’s moulded


surface)
Showing a collection of waterlines at various drafts. Each line represents the
shape of a waterplane. The load waterplane (LWP), or load waterline (LWL),
represents the waterplane at which the ship is designed to float in loaded
condition. A design waterline (DWL) is the waterplane for which the ship is
designed. This does not mean the ship cannot operate at any other draft, but that
the designer tried to optimise the performance of the ship for that draft. This, as
you can imagine, is often the same as the LWL.

Body plan (intersection of transverse vertical planes with ship’s moulded


surface)
A collection of transverse sections at regular, designated stations along the ship’s
length. For normal purposes, only one half of the section shape is shown. The
forward half of the ship is shown on the right hand side of the centreline and the
aft half is shown on the left hand side. The stations, normally numbered 0 to 10,
with stem and stern regions having more stations and around the parallel middle
body at intervals of 1. For example, a basic design lines plan may have stations
0, ¼, ½, ¾, 1, 1½, 2, 3, 4, 5, 6, 7, 8, 8½, 9, 9¼, 9½, 9¾, 10. Station 0 is known as
the forward perpendicular, and is the plane perpendicular to the centreplane and
the waterplanes passing through the intersection between LWL and the forward
edge of the stem. Station 10 is the aft perpendicular and positioned at the aft side
of rudder post or centreline of the rudder stock. Sometimes extra stations are
placed outside these two perpendiculars. Station 5 corresponds to midships.

Sheer plan or profile(intersection of longitudinal vertical planes with ship’s


moulded surface)

4
Shows longitudinal sections of the hull on the planes parallel to the centreplane at
a number of offsets. The forward lines are known as bowlines, while the aft lines
are called buttock lines or buttocks. The line at 0 offset will show the profile
outline of the vessel together with the deck at centre line. The plan also shows
deck at side line.

A lines plan is usually a very long drawing, and to reduce the inconvenience of
having to deal with such long drawings, it is sometimes drawn with the forward
half superimposed on the aft half.

In order to ensure fair hull form some section shapes of longitudinal planes other
than the two orthogonal planes are also drawn below the centreline of the half-
breadth plan. These are known as diagonals, and are of particular significance for
sailing yacht forms, as they often sail with a static heeling angle, and, therefore,
fairness of the lines at heel is crucial for their performance.

(b) Table of Offsets

Offsets are transverse distances from the centreplane to the inside of the shell
(moulded offsets). These are arranged in a tabular form with columns for stations
and rows for waterlines, or the other way around. In addition, other information,
such as sheer, may also be included.

(c) Form Coefficients

There are a number of non-dimensional coefficients which are very often used to
indicate the major geometric characteristics and to give an idea of the performance
of the vessel. For geometrically similar vessels the form coefficients will b
identical.

Block coefficient (CB)


This is fraction of the volume of displacement of the underwater form ( ∇ )
occupying the rectangular block whose sides are equal to LBP, B and T. Thus,


CB =
L BP × B × T

The block coefficient gives an indication of the fullness of the form and its
value may vary from 0.38 for some high speed vessels, yachts and military
vessels through 0.60 for passenger ships and aircraft carriers to 0.88 for large
crude oil carriers.

Midship section coefficient (CM)


This is the ratio of the immersed area of the midship section (AM) to the area of
the enclosing rectangle whose sides are equal to B and T. Thus,

AM
CM =
B ×T

This coefficient expresses the fullness of the midship section and its value is
usually around 0.75 to 0.98. For extreme forms it may be as low as 0.65 or as
high as 1.0.

5
Prismatic coefficient (CP)
This is the ratio of the volume of the underwater form to the volume of a prism
having a constant cross section of the immersed midship section and length
equal to LBP. Thus,


CP =
AM × L BP

This coefficient is sometimes called longitudinal prismatic coefficient to


distinguish it from the vertical prismatic coefficient (see below). It is a very
important parameter determining the forward motion resistance of the vessel.
As can be expected, the ships with lesser resistance will be finer in form and
therefore will have smaller CP. Its value may range from about 0.55 to 0.80.

Vertical prismatic coefficient (CVP)


This is the ratio of the volume of the underwater form to the volume of a prism
having a depth equal to the vessel’s mean draft (T) and a constant horizontal
cross section identical to the waterplane at that draft. Thus,


CVP =
AW × T

where AW is the waterplane area.

Waterplane area coefficient (CW)


This is the ratio of the waterplane area to its circumscribing rectangle. Thus,
AW
CW =
LWL × B

Unless otherwise stated, CW is for the load waterline of the vessel, at which LWL
is often the same as LBP.

Table 1.1 gives sample principal dimensions and/or some form coefficients. Perhaps
you would like to calculate the items left blank and compare them between various ship
types.

Table 1.1 Examples of principal dimensions


Tug Ferry Cargo Cargo Passenger Tanker Tanker
LPP (m) 31.7 47.2 126.5 161.0 201.0 125.0 250
B (m) 8.50 13.25 18.9 23.2 28.5 18.9 34.25
T (m) 3.50 2.45 8.0 9.0 9.9 7.3 14.1
∆ (tonnes) 388 888 12,480 21,340 35,590 13,680 101,500
AM (m2) 20.50 30.25 144.7 206.2 275.0
TPC (tonnes/cm) 1.99 4.68 19.60 28.10 43.85 20.70 77.50
CB
CP
CVP
CW
CM 0.99 0.99

6
1.3. FLOTATION

(a) Archimedes’ Principle

Most people have heard usually at school the funny but exhilarating story of
Archimedes running through the street stark naked shouting ‘Eureka!’ We all
experience lightening of our body when we get into a bath and therefore it was not
this phenomenon itself that he discovered. The significance of his discovery is in
quantifying that experience precisely, thereby enabling the application of this
principle in ‘engineering’. It is so fundamental to naval architecture, that it is
necessary to examine his principle with the help of more modern understanding.

When a body is immersed in a fluid, the body will experience an upward force
equal and opposite to the weight of the fluid displaced by it. This is known as the
Archimedes’ Principle and the upward force is called buoyancy. We now know
that the buoyancy is the resultant of all the normal pressures exerted by the fluid on
the entire immersed surface of the body. This pressure is known as hydrostatic
pressure and is proportional to the depth of immersion, or

P = ρgd
where ρ = mass density of the fluid
g = gravitational acceleration (=9.806 m/s2)
d = depth of immersion of the point.

For an arbitrary body floating in static equilibrium in a fluid shown in Fig.1.4, the
vertical force (or the buoyancy force) is the sum of the vertical component of the
surface pressure, or

B = ∫ P cosθ ⋅ dA = ∫ ρgh ⋅ dA ⋅ cos θ


A A

It is evident that h ⋅ dA ⋅ cos θ is in fact the immersed volume of the column which
presents the surface dA to the fluid at an angle θ at its end. Therefore,
∫ h ⋅ dA ⋅ cosθ is the total immersed volume of the object. Let the volume be V,
A
then

B = ρgV
provided ρg is constant, which is practically true for incompressible fluid and for
the kind of depth we are concerned with. This proves the crucial Archimedes’
discovery.

One can also prove easily that the sum of horizontal component of the pressure is
zero.

7
W L

W L

h
ds

(a) Hydrostatic pressure on the immersed surface (b) An elemental volume

Fig.1.4 Mechanism of buoyancy force

(b) Notation

The volume of the ship underwater is known as ‘volume displacement’ and is


represented by the symbol ∇ . The normal unit for volume displacement is m3.
The mass of the water displaced is called ‘mass displacement’ and is written ∆.
Thus, ∆ = ρ∇ . The unit of mass displacement is usually tonnes (the mass of 1 m3
of fresh water is 1 tonne, that of 1 litre is 1 kg). Note that ∆ is the same as the mass
of the ship and g∆ (sometimes called force displacement) will be the same as the
weight of the ship. Indeed one hardly ever sees any special symbol for the ship’s
weight, since it can be represented by the displacement. The unit of force
displacement is N, but since mass is in tonnes, it is more convenient to use kN
(weight of 1 tonne is approximately 9.806 kN).

(c) Static Equilibrium

Although the buoyancy force is distributed all over the ‘wetted surface’, it is often
more convenient to consider that the total buoyancy force acts upwards from a
single point. This is the point about which the moment of the hydrostatic pressure,
when summed over the entire wetted surface, becomes zero. It is called the centre
of buoyancy. In a similar manner the gravitational force (i.e. the weight of the
object) can also be regarded to act from one point: this is the centre of gravity.

In general, a state of equilibrium exists if and only if

(a) the sum of all the forces is zero; and


(b) the sum of all the moments is zero.

An object freely floating in still water, therefore, will have just the right volume
immersed to provide exactly the same buoyancy force as its weight, and will take
an attitude which puts the centre of gravity vertically in line with the centre of

8
buoyancy (regardless of vertical relative position). This state is known as static
equilibrium in contrast to dynamic equilibrium which will be discussed later. Of
course, if its total volume cannot provide sufficient buoyancy, it will simply sink.
If the total volume can produce buoyancy exactly the same as the weight, it will be
‘neutrally buoyant’, as submarines can be in certain conditions.

If the first of the two equilibrium conditions is not met, then the object will either
sink further or rise until that condition is fulfilled. If only the second condition is
not met, then the object will rotate itself until the equilibrium is reached. In
general, however, these two processes occur at the same time.

Unlike objects of simple geometric shapes, the volume displacement of a ship for
any given draft is not simple to calculate. These days, all that can be done by a
computer at a blink of an eye, and thus presents no problem. However, in the days
when every single computation had to be done by hand, it was immensely
important to devise methods of instantly reckoning the expected draft for a given
loading condition or the mass displacement for a given draft. Naval architects
developed many different curves and other ‘hydrostatic’ items for this purpose, and
we shall examine some of the more important ones here, as, despite the modern
technology, these are still used extensively.

(d) Curve of Displacement

The volume displacement calculated for a range of draft can be plotted against
draft, and the curve is smoothed (naval architects are, if nothing else, past masters
of generating smooth curves). From this curve, the volume displacement at any
mean draft can be readily obtained and vice-versa. This curve presupposes that the
ship will have a fixed pre-defined longitudinal attitude (trim). Trim, however, is
the subject of a later section.

(e) Tonnes Per cm Immersion (TPC)

The loading condition of a ship changes often and it is important to keep track of
what draft the ship will float at the end of a minor loading/unloading operation. For
this the curve of displacement may be used, but for obtaining a quick indication of
the change of draft for a given change in mass displacement, a ratio known as
tonnes per cm immersion is very useful. The TPC of a ship at any given draft is the
mass required to increase the draft by one cm (parallel to the existing waterline, and
hence the terms ‘parallel sinkage’ and ‘parallel rise’).

Consider a vessel floating at a draft T in water of density ρ (by the way, 1.025 or
1.026 tonnes/m3 is often taken for the normal mass density of sea water). Its
waterplane area is AW at this draft. Some cargo is now placed on the vessel so that
it sinks by 1 cm. This increases the underwater volume by approximately

d∇ = AW × 1 cm
= AW × 0.01 (in m 3 , if AW is in m 2 )

Therefore, the increase in mass displacement is

d∆ = d∇ × ρ = 0.01ρAW

Note that this increase in displacement will be identical as the mass of the cargo
loaded which caused the parallel sinkage.

9
Conversely, we can find the parallel sinkage or rise caused when a mass m is loaded
or unloaded as follows

parallel sinkage = m / TPC

A moment’s reflection will show that TPC is in fact equivalent to the slope of the
curve of displacement. Note also that in the above discussion we have assumed the
layer of the body which is immersed by increased mass of the object has a constant
waterplane area. Strictly speaking, this is not true, but most ships are virtually
‘wall-sided’, and it is accurate enough for most engineering purposes for small
changes in draft. Indeed this type of approximation, simplification and linearisation
used to be essential before the advent of computers. Remember that computers
began their serious inroads into ship design only in 1970s!

(f) Effect of Water Density

Ships travel, and the seas of different areas have different mass density, due to
changes in salinity and temperature. A more pronounced change in mass density of
water is experienced by ships on moving from fresh water into sea water (as in the
Great Lakes and St Laurence Seaways). There is about 2.5% difference in the
density of fresh water and sea water, and obviously this will cause sinkage or rise of
the vessel during the transit. In all these cases, only one principle should be
remembered: ‘the mass displacement of the ship should be the same as the mass of
the ship at all times, whatever the density of water.’ This means change in the
underwater volume which can only be achieved by altering the draft.

Consider a vessel of displacement 15,000 tonnes floating in sea water of density


1.025 tonnes/m3 at a draft of 7m . The volume displacement in sea water is

∇ S = 15000 / 1.025 = 14634 m 3

When this ship moves into fresh water of density 1.0 tonne/m3, its volume
displacement will be

∇ F = 15000 / 1.0 = 15000 m 3

The increase in volume displacement, 366 m3, can only be achieved by the
(parallel) increase of draft. If the waterplane area of the ship is 2000 m2 at 7 m
draft, then the parallel sinkage is

dT = 366 / 2000 = 0.183 m or 18.3 cm

Note that in general, the new waterline will not necessarily be parallel to the
original waterline, but it is convenient to consider the parallel rise or sinkage first
and then the change in trim separately.

10
TUTORIAL No.1

1. A hollow steel cylinder is 1.83m in diameter, 7.62m long and weighs 16.26 tonnes. At
what draft will it float in sea water ( ρ = 1.025 tonnes/m 3 ), when its axis is vertical?

What will then be the hydrostatic pressure at any point on the bottom?

2. When a ship, without changing weight, passes from sea water to fresh water, she will sink
further in the water, because fresh water is less dense than sea water. Show that the
increase in draft, say d (in cm), is given by

(
∆ ρs − ρ f )
d=
ρ f ⋅ TPC

where ∆ = displacement of the ship in tonnes


ρ s = density of sea water in tonnes/m3
ρ f = density of fresh water in tonnes/m3
TPC is for sea water.

3. Show that for any ship

CP C
= W
CVP C M

4. A ship, 122m long, has a beam of 15.25m and floats in sea water with an even keel draft of
5.50m. If the block coefficient, CB, is 0.695, what is the displacement?
The immersed midship section area is 82.50m2. Calculate the values of CP and CM.

5. A ship has the following particulars:

L = 128m, B = 19.20m, T = 8.85m, CB = 0.713, CM = 0.945.

Calculate

- Moulded (mass) displacement in sea water


- Area of immersed midship section
- Prismatic coefficient.

6. A ship of displacement 1747.0 tonnes has a beam-to-draft ratio equal to 3.53 when floating
in sea water. If the immersed midship section is 30m2, CB = 0.537 and CM = 0.834,
calculate the length, beam and draft of the ship.

11
1.4. FUNDAMENTAL MOMENT THEORIES AND THEIR APPLICATIONS

One of the most important concepts in engineering is moments, of various order, of


lines, areas, volumes, masses, forces and, even, moments. Indeed, the area of a 2-D
shape can be called its 0th moment (the lever is brought up to the power of 0). Of these,
the first and second moments are encountered the most often in naval architecture.
Here we shall confine our discussion to the first and second moments of area, volume,
force and mass.

(a) Area

The area of an arbitrary 2-D shape in the xy-plane shown in Fig.1.5 is calculated by
first considering a thin strip of width dx. The length of this strip is η = y1 – y2, and
thus its area is ηdx. Therefore,

x2
A = ∫ ηdx
x1

When it is not possible to express η in a mathematical form, this integration can be


performed numerically (using e.g. Simpson’s rules).

y y
y1

x
dx

y2 A
x x

Fig.1.5 Calculation of area Fig.1.6 Calculation of first moments

(b) First Moment of Area

The first moment of an infinitesimal area about a given axis is the area multiplied
by the distance between that axis and the area (lever). The word ‘infinitesimal’ is
used here to signify that the area is so small that the lever from the reference axis to
that area can easily be determined. This is analogous to the moment of force, often
known simply as moment, where the force is multiplied by the lever.

Consider an arbitrary, enclosed 2-D shape of area A shown in Fig.1.6. If we take a


very small part of it, and let the area dA and the lever to y-axis x. Then the moment
of this element about y-axis is xdA. Therefore, the moment of A about y-axis is

12
M y = ∫ x ⋅ dA
A

The symbol ∫ means integration over the whole area A. Similarly


A

M x = ∫ y ⋅ dA
A

The lever can be positive or negative. It follows, therefore, that a first moment of
anything can be positive, zero or negative. Indeed it can be seen at once that the
first moment of an area about the axis of symmetry (if it exists) is zero.

Furthermore, in Fig.7, My is obviously positive, since all the levers are positive;
similarly Mx is also positive. Consider now another extreme case where the y-axis
is entirely on the right hand side of the area and the x-axis is well above the area.
Then, both moments will be negative, since the levers are always negative. This
means that as the y-axis moves from the left hand side of the area to the right, it
will pass a point at which My is zero – keep the y-axis there. Similarly as the x-axis
moves upwards from below the area, it will pass a point at which Mx is zero, and we
keep the x-axis there. The intersection of these two axes is known as the centroid
of the area.

The definition of the centroid of an area, therefore, is that it is the point of


intersection of any two axes, the first moments of the area about which are zero.

Consider an arbitrary 2-D area A and two parallel axes distance tg apart, say y and
y1. Then the first moments of the area about these axes are

M y = ∫ x ⋅ dA and
A

( )
M y1 = ∫ x1 ⋅ dA = ∫ x + t g dA = ∫ x ⋅ dA + ∫ t g ⋅ dA
A A A A

= M y + t g ∫ dA = M y + t g ⋅ A
A

If we now assume that the axis y passes through the centroid, then

My =0

Therefore

M y1 = t g ⋅ A

In other words the first moment of an area can be obtained by multiplying the area
with the distance from the centroid to the axis. Conversely, the position of the
centroid relative to any reference axis can be found by dividing the first moment of
the area about that axis by the area. Actually these two principles are applicable to
any kind of first moments, and are used in determining centre of gravity, centre of
buoyancy, centre of flotation, and so on.

13
Centroid of Waterplane

Identifying the centroid of waterplane for a given draft is very important, as it is


required to estimate the longitudinal stability and trim. Normal ships have port-
and-starboard symmetry. Therefore, it is obvious that the centroid of the
waterplane will be on the centreline. It remains to identify the longitudinal location
of the centroid, and this can be done for any reference point, but normally it is done
for the midship. The longitudinal distance from the midship to the centroid of
waterplane is known as LCF (standing for Longitudinal Centre of Flotation), the
reason for which will be discussed later on.

Fig. 1.7 A waterplane with the axis system

Consider a waterplane shown in Fig.1.7. The origin of the axis system is on the
waterline amidships, so that LCF can be estimated relative to the midship directly
without any adjustment.

The first moment of the WP about y-axis is

Lf
My =2 ∫ xydx
− La

where La and Lf are the length of aft body and fore body respectively
y is the half breadth of the waterplane (a function of x).

The area of the WP is

Lf
AW = 2 ∫ ydx
− La

Therefore, midship to centroid is

Lf

My
∫ xydx
− La
x= = LF
AW
∫ ydx
− La

14
Note that, in this notation, a positive LCF indicates the centroid is forward of the
midship. Most often longitudinal positions of all kinds are given as ‘aft or fwd of
midship’ rather than as signed numbers. Nevertheless, having a sign convention is
invaluable in many calculations.

(c) Second Moment of Area

The second moment of an infinitesimal area about a given axis is the area
multiplied by the distance between the axis and the area (lever) squared. A second
moment of area is not as easy to visualise as the first moment, but it is analogous to
a second moment of mass, which is also known as mass moment of inertia.

The second moment of an area about Ox-axis is often represented by Ix or Ixx, and it
is in fact a moment about x-axis of the first moment of the area about x-axis. Note
that one can have a moment about y-axis of the first moment of the area about x-
axis, which is known as ‘product of area’ and represented by a symbol Ixy.

Consider an arbitrary area of Fig.1.8. In this case

I x = ∫ y 2 dA and I y = ∫ x 2 dA
A A

For example, for the rectangle shown in Fig.1.9, second moment of an elemental
strip about y-axis is x 2 bdx . Therefore

a a
⎡1 ⎤ 1
I y = ∫ x bdx = ⎢ x 3 b ⎥ = a 3 b
2

0 ⎣3 ⎦0 3

Similarly, it can be shown that

1
I x = ab 3
3

Fig. 1.8 an arbitrary area on x-y plane Fig. 1.9 A rectangle on x-y
plane

Parallel Axis Theorem

Consider an arbitrary area shown in Fig.1.10. y-axis passes through the centroid of
the area, and an η-axis is parallel to it at a distance l away.

15
Fig.1.10 Parallel axis theorem

Then

I y = ∫ x 2 dA and
A

( )
I η = ∫ ( x + l )2 dA = ∫ x 2 + 2 xl + l 2 dA = ∫ x 2 dA + ∫ l 2 dA + 2 ∫ xldA
A A A A A

Since l is a constant,

I η = ∫ x 2 dA + l 2 ∫ dA + 2l ∫ xdA = I y + Al 2 + 2lM y
A A A
where A is the area of the shape and My is first moment of the area about y-axis.

However, since y-axis passes through the centroid, My = 0. In other words,

I η = I y + Al 2

This means that the second moment of an area about any axis is equal to the second
moment of that area about an axis parallel to that axis passing through the centroid of
the area plus the area multiplied by the distance squared. This applies equally to
second moment of mass and volume as well. It is a very important principle in many
branches of engineering.

In particular, the second moment of a waterplane area about the transverse axis
passing through its centroid (i.e. centre of flotation) is known as the longitudinal
second moment of waterplane area, or IL, and that about the centreline is referred to as
the transverse second moment of waterplane area, or IT. We shall deal with practical
applications of these items later on.

Example 1

Calculate the second moment of area of a rectangle (b x h) about an axis passing


through the centroid and parallel to the b-side. Then calculate the second
moment of this rectangle about the b-side.

Put a Cartesian co-ordinate system with the origin at the centroid of the rectangle
and the x-axis parallel to the b-side and the y-axis parallel to the h-side. Consider
a thin strip of width dy, lengthwise parallel to the x-axis and at a distance y away
from the x-axis. Its area is bdy and its second moment about the x-axis is

16
dI x = y 2 bdy

Therefore the second moment of the whole rectangle about the x-axis is

h2 h/2
⎡1 3 ⎤ bh 3

2
Ix = y bdy = ⎢3 y b ⎥ =
−h 2 ⎣ ⎦ − h / 2 12

The distance between the x-axis and the b-side is h/2. Using the parallel axis
theorem,

2
⎛h⎞ bh 3 bh 3 bh 3
I b = I x + bh⎜ ⎟ = + =
⎝2⎠ 12 4 3

Example 2

A right angle triangle has two shorter sides of length b and h. Find the second
moment of this triangle about the side of length b. Using this result, calculate its
second moment about an axis parallel to that side, but passing through the
centroid of the triangle.

Put the x-axis along the b side, and the y-axis along the h side. Then the
hypotenuse can be expressed as

h b
y=− x + h or x = b − y
b h

There are two ways of going about it. The first method takes a narrow strip of
width dy, lengthwise parallel to and at a distance y from the x-axis. Its area is

⎛ b ⎞
xdy = ⎜ b − y ⎟dy
⎝ h ⎠

Its second moment about x-axis is

⎛ b ⎞ ⎛ y 3 ⎞⎟
dI x = y 2 ⎜ b − y ⎟dy = b⎜ y 2 − dy
⎝ h ⎠ ⎜ h ⎟⎠

Thus the second moment of the area is

h⎛ h
y 3 ⎞⎟ ⎡ y3 y4 ⎤ bh 3
I x = b∫ ⎜ y 2 − dy = b ⎢ − ⎥ =
⎜ h ⎟⎠ 4h ⎥⎦
0⎝ ⎣⎢ 3 0
12

We may also take the strips of width dx, parallel to the y-axis. In this case the
strip may be considered to be a rectangle. Its area is ydx and its second moment
about the x-axis is (from Example 1)

17
3
y 3 dx 1 ⎛ h ⎞
dI x = = ⎜ − x + h ⎟ dx
3 3⎝ b ⎠

When this is integrated along the x-axis from 0 to b, the same result as above is
obtained.

Now, the second moment about the centroidal axis can be found by using the
parallel axis theorem (remembering that the least second moment of an area is
about its centroidal axis)

2
1 ⎛h⎞ bh 3 bh 3 bh 3
I xx = I x − bh⎜ ⎟ = − =
2 ⎝3⎠ 12 18 36

Example 3

A twin pontoon type semi-submersible has four columns of diameter 12m. The
transverse distance between the centres of the columns is 70m. Calculate the
transverse second moment of the waterplane area in its operational condition.
Ignore all bracings. The second moment of area of a circle of diameter d about
πd 4
its diametrical axis is .
64

Semi-submersibles are designed to operate so that only its columns and


associated structures (such as bracings) pierce the water surface. The area of the
circle is πd 2 / 4 . The distance from the centre of the circle to the longitudinal
axis of the semi-submersible is 70/2 = 35m. The second moment of the circle
about the longitudinal axis (centreline) can be found by using parallel axis
theorem

πd 4 πd 2
IT = + 35 2 = 1017.88 + 138,544.24 = 139,562.12 m 4
64 4

Since all four columns are of identical size and at the same distance away from
the centreline,

I T = 4 × 139,562.12 = 558,248.48 m 4

It can be seen that the contribution of the second moment about its own
diametrical axis is very small (less than 1%) indeed. Nevertheless, the second
moment is comparable to a monohull vessel despite its small waterplane area
thanks entirely to the separation. As will be seen later, the transverse second
moment of the waterplane is one of the main factors which influence the stability
of a vessel. This explains why catamarans and semi-submersibles have very
good initial stability characteristics.

18
TUTORIAL No 2

1. Find the first moments of area about x and y axes for the various shapes shown
below, and determine the location of their centroid.

y
y

(6, 10) (2, 5) (4.5, 5)

(2, 4)
(1, 1) (6, 1)
x x
(9, -1)

y y

(-2, 5) (6, 5)

8
30
°
(2, 1)
30°
x x

(6, -1)
(1, -1.5)
(-2, -2) (4, -2)

2. For the waterplane shown below calculate

first moment of area about midship and hence LCF from midship;
longitudinal and transverse second moments of area (about axes through the centroid).

20 45 35 30
20

All dimensions in m.

19
3. Calculate the longitudinal and transverse second moment of area of a catamaran shape
below.

5m
15m

7m
2m

4. An old pentagon-type semi-submersible has 5 columns of 15m diameter equally spaced


around a circle of radius 30m. Calculate the second moments of the waterplane of this
semi-submersible.

20
1.5. PROPERTIES OF 3-D SHAPES

The first and second moments of area are usually about an axis. The first moment of
volume and mass, on the other hand, is calculated about a plane. Their second moment,
however, is calculated about an axis, as it is the inertia of the object in rotational motion
about that axis (hence the alternative name, mass moment of inertia). Despite these
little differences, the properties of volume and mass are really the same as those of 2-D
shapes.

For instance, with an arbitrary object in space, the vertical mass moment about x-y
plane is

M xv = ∫ zdm
M

where z is the vertical distance from the reference plane (x-y plane in this case) to
the elemental mass dm.
M denotes the whole mass.

Therefore the centroid of the object is at a point

M xy ∫ zdm
z= = M

M M

above the reference plane. The same principle applies in the other directions. Note that
volume can be treated in exactly the same way as mass. This is how the centres of
gravity and buoyancy are calculated. For example, the centre of buoyancy can be
found in the following manner.

Consider a Cartesian co-ordinate system where the origin is fixed at a point amidships
on the centreline on the water plane as shown in Fig. 12. Positive x is forward, y is
positive starboard and z is positive upward, maintaining the right-hand rule. Then the
moments of the underwater volume about the x-y and y-z planes are

M xy = ∫ zdv and M yz = ∫ xdv


V V
where V is the whole underwater volume and dv is the elemental volume.

M yz
The longitudinal position of the centre of buoyancy (LCB) is from the midship
V
M yz
and its vertical position (VCB) is from the waterline, remembering the sign
V
convention.

The mass moment of inertia of an object is calculated about an axis, as mentioned


before. For example the moment of inertia about x-axis is

∫ (y )
2
I xx = + z 2 dm
M

21
( )
Note that y 2 + z 2 is the squared distance of dm from the x-axis.

Movement of Centroid Due to Addition or Subtraction of Mass or Volume

Consider an object, with mass m1 and the centroid l1 from a reference plane. Some
mass m2 is added to it and its centroid is at l2 from the plane. Then the distance of the
new centroid from the reference plane, l, is

m1l1 + m2 l 2
l=
m1 + m2

This is quite obvious from the principles that


- The distance to the centroid can be obtained by dividing the first moment by the
mass; and
- The moment of an object about an axis can be found by summing the moments of
its component parts about the axis.

Of course, when m2 is subtracted from m1, then the sign of m2 will be negative.

Sometimes it is more convenient to know how much the centre of gravity moves in
this situation. The movement of the centroid (from l1, that is), dl is

m1l1 + m2 l 2 m l + m2 l 2 − m1l1 − m 2 l1 m 2 l 2 − m2 l1 m2 (l 2 − l1 )
dl = l − l1 = − l1 = 1 1 = =
m1 + m 2 m1 + m 2 m1 + m 2 m1 + m2

The numerator of the last is the mass moment of m2 about the plane passing through
the centre of gravity of m1 and parallel to the original reference plane.

This idea can be applied to the situation where parts of a given object are moved to
new locations. This case can be regarded as some parts removed and the same parts
added elsewhere. Some worked examples will illustrate this very much more clearly.

Example 4

A ship of mass displacement 10,000 tonnes has its centre of gravity at 7m above
baseline. Some cargo of mass 1000 tonnes is loaded onto the ship so that its centre of
gravity is at 10m above baseline. Calculate the new vertical position of the ship’s
centre of gravity (VCG).

10000 × 7 + 1000 × 10
The new KG = = 7.273m
10000 + 1000

(We may get the same result by taking the moment about the original centre of
gravity, thus

10000 × 0 + 1000 × (10 − 7 )


the new CG relative to the original CG = = 0.273m )
10000 + 1000

22
Example 5

A ship of mass displacement 8,000 tonnes has its centre of gravity at 4m aft of
midship. Some cargo of mass 500 tonnes is moved forward by 20m. Calculate the
position of the longitudinal centre of gravity (LCG).

We may regard the movement as a combination of two operations: unload 500 tonnes
from a location x, say, and load 500 tonnes at a location x + 20m. Taking fwd of
midship as positive,

The new LCG =


8000 × (− 4) − 500 × x + 500 × ( x + 20) ) 8000 × (− 4) + 500 × 20
= = −2.75m
8000 − 500 + 500 8000

I.e. 2.75m aft of midship. The important thing to note here is that when a mass is
simply moved, we do not need to do this in two operations. All we have to do is add
the moment of the movement. It is important to note in this case, however, that the
total mass has not changed.

Example 6

A ship of mass displacement 10,000 tonnes has its centre of gravity at 8m above
baseline. The following operations are carried out:

Load 500 tonnes 3m AB


Discharge 300 tonnes 5m AB
Load 200 tonnes 12m AB
Move 1000 tonnes downward by 2m

Calculate new VCG.

10000 × 8 + 500 × 3 − 300 × 5 + 200 × 12 − 1000 × 2


The new VCG = = 7.731m
10000 + 500 − 300 + 200

In fact when a number of operations are carried out like this, it is best to compile a
table, something like this

Operation Mass (tonnes) v.c.g. (m) d∆ Moment


Original ship 10000 8 10000 80000
Load 500 3 +500 1500
Discharge 300 5 -300 -1500
Load 200 12 +200 2400
Move 1000 by -2 0 -2000
Σ 10400 80400

80400
New VCG = = 7.731m
10400

23
PART 2 NUMERICAL INTEGRATION TECHNIQUES

2.1. INTRODUCTION

Not surprisingly integration in various forms keeps appearing in hydrostatics, since area
is calculated by integrating line lengths and volume by integrating sectional areas and
so on. It is hardly an exaggeration to say that most of hydrostatic calculations were
based on moment theories and integration of one sort or another.

When we are dealing with geometrical shapes which can be readily expressed in a
mathematical form (particularly polynomials), the integration in most cases can be
accomplished by manipulating the mathematical expression. Ship hulls are difficult to
represent in a convenient mathematical expression (the piecewise polynomials often
used to express hull forms are in general more difficult to integrate). In any case, naval
architects of bygone era before computers arrived had to devise some methods to do the
integration. They developed a number of numerical integration techniques which are
used extensively even now. All these numerical integration produces approximate
answers, although sufficiently accurate for engineering purposes. Indeed ingenious,
albeit somewhat clumsy to use requiring infinite care and patience, devices called
‘integrators’ were invented to carry out ‘accurate’ integration, but thankfully these have
disappeared from ship design offices, as computers can produce more accurate results
in a much shorter time.

In this part, we will explore some of the more popular numerical integration techniques.
Before doing so, however, it will be useful to summarise various ways of finding some
hydrostatic quantities.

To get Integrate Direction


WP area breadths longitudinally
Section area breadths vertically
WP area vertically
volume
section area (SA) horizontally
displacement
vertical moment of SA longitudinally
vertical moment
vertical moment of WPA vertically
of displacement
long’l moment of SA longitudinally
long’l moment
long’l moment of WPA vertically
of displacement
IT of WP transverse second moment of longitudinally
breadths
IL of WP longitudinal second moment of longitudinally
breadths

2.2. TRAPEZOIDAL RULE

Consider a curve A-D as shown in Fig.2.1(a) with the known points of (xi, yi), i = 0, 1,
2, 3 ... n, and we would like to calculate the area ABCD. Note that point (x0, y0) is at A
and (xn, yn) is at D. The curve A-D is assumed to be made of n straight line segments.

24
Then an approximate area of ABCD can be found by simply summing the areas of the
trapezoids thus formed as shown in Fig.2.1(b).

y2 y3
y1 yn-2 y
D n-1 D
A A yn
y0

x0 xn
B C B x1 x2 x3 .. x n-2 C

Fig.2.1(a) Area to be calculated Fig.2.1(b) Simplification to trapezoids

Thus area ABCD =

1
[(x1 − x0 )( y 0 + y1 ) + (x 2 − x1 )( y1 + y 2 ) + ...... + (x n − xn−1 )( y n−1 + y n )]
2

The accuracy of this technique depends largely on how closely the curve can be
represented as a collection of straight lines. It is obvious therefore the more points on
the curve are known, the more accurate the result will be. The method is extremely
simple to understand and use, and despite its somewhat crude approximation can often
produce sufficiently accurate results. Indeed some very sophisticated computer
programs sometimes rely on this method.

Trapezoidal rule is one of the Newton-Cotes formulae for numerical integration which
cover polynomials of any order, trapezoidal being for linear polynomial approximation.
For any curve other than straight line, approximating it with a higher order curve will
produce more accurate area, and for this we turn to Simpson’s rules which are again
special cases of Newton-Cotes formulae.

2.3. SIMPSON’S RULES

Simpson’s rules are the most widely used numerical integration techniques because of
their simplicity and wide applicability. Consider a curve A-D as shown in Fig.2.2(a)
with three known points the x values of which are such that x2 – x1 = x3 – x2 (in other
words evenly spaced). For ease of calculation we will put the y axis at x = x2, thus
making
x2 = 0. Let x1 = – h, then x3 = h.

We now assume that the curve A-D can be expressed as a cubic polynomial of x, thus

y = a 0 + a1 x + a 2 x 2 + a 3 x 3 (2.1)

25
y

y2
D F
A A y4
y1 y3 y1
E y5
y2 y3

x1 O x3
x
x1 x2 x3 x4 x5
B x2 C B C D

Fig.2.2(a) Simpson’s First Rule Fig.2.2(b) Five equally spaced ordinates

The area ABCD, say A, can be found as

h
h ⎡ 1 1 1 ⎤ 2
A = ∫ ydx = ⎢a 0 x + a1 x 2 + a 2 x 3 + a3 x 4 ⎥ = 2a 0 h + a 2 h 3 (2.2)
−h
⎣ 2 3 4 ⎦ −h 3

Since

y = y 2 when x = 0 , a 0 = y 2 , and (2.3)


y = y1 when x = −h , y1 = a 0 − a1 h + a 2 h − a 3 h
2 3
(2.4)
y = y 3 when x = h , y1 = a 0 + a1 h + a 2 h + a 3 h
2 3
(2.5)

Adding (2.4) to (2.5) and solving the result for a2 gives

1
( y1 + y3 ) − y 2
a2 = 2
h2

Substituting this into (2.2) gives

⎛1 ⎞
( y + y3 ) − y 2
2 3 ⎜⎜ 2 1 ⎟ h
area ABCD = 2 y 2 h + h ⎟ = ( y1 + 4 y 2 + y 3 )
3 ⎜ h2 ⎟ 3
⎜ ⎟
⎝ ⎠

This is known as Simpson’s First Rule and a moment’s reflection will show that as
long as three points on the curve, the x-values of which are evenly spaced, are known,
this rule can be applied regardless of what those x-values are (remember we derived the
above for x2 = 0, but x-values do not appear in the rule except the ‘common interval’).
This rule is sometimes known as Simpson’s 1/3 rule or 1-4-1 rule for obvious reasons.

26
Note that this rule produces a precise value for any polynomial up to cubic. Therefore,
the accuracy of this method depends largely on how closely the curve can be
approximated as a cubic polynomial. If, on the other hand, the curve contains segments
which can only be expressed in polynomials of different orders, for example a cubic
curve adjoining a straight line segment, the accuracy will decrease.

If we have to deal with 5 points on the curve with the x-values equally distributed as
shown in Fig.2.2(b), then we can divide the area into two segments: ABCE and ECFD
and apply Simpson’s first rule to each portion, thus

area ABFD =
h
( y1 + 4 y 2 + y 3 ) + h ( y 3 + 4 y 4 + y5 ) = h ( y1 + 4 y 2 + 2 y 3 + 4 y 4 + y 5 )
3 3 3

The numbers 1, 4, 2, 4 and 1 are known as Simpson multipliers.


If there is a half station at the beginning – for example, 0, ½, 1, 2, 3, 4, 5, the integration
can be performed by applying the rule separately to the segments 0, ½, 1 and 1, 2, 3, 4,
5. Of course the common interval for the first segment will be half of that for the latter.
Therefore, the Simpson multipliers in this case will be 0.5, 2, 1.5, 4, 2, 4, 1. One often
encounters Simpson integration in a tabular form and a sample of this is given in Table
2.1.

Simpson’s First Rule, as we have seen, applies to three equally spaced ordinates. There
is another Simpson’s rule which applies to four equally spaced ordinates, and it is
known as Simpson’s Second Rule (sometimes known as Simpson’s 3/8 rule or 1-3-3-1
rule). With the four ordinates y1, y2, y3 and y4 at x1, x2, x3 and x4, and the common
interval h,

3h
area = ( y1 + 3 y 2 + 3 y 3 + y 4 )
8

You can work out how this may be so and prove that it produces accurate results for up
to cubic polynomials quite easily.

Table 2.1 An example calculation of area using Simpson’s first rule

Station Ordinates SM f(A)


0 10.00 0.25 2.50
¼ 10.61 1 10.61
½ 11.30 0.5 5.65
¾ 12.07 1 12.07
1 12.94 0.75 9.70
1½ 14.92 2 29.84
2 17.25 1.5 25.88
3 22.94 4 91.75
4 30.00 2 60.00
5 30.00 4 120.00

27
6 30.00 2 60.00
7 19.13 4 76.50
8 10.50 1.5 15.75
8½ 7.03 2 14.06
9 4.13 0.75 3.09
9¼ 2.88 1 2.88
9½ 1.78 0.5 0.89
9¾ 0.82 1 0.82
10 0.00 0.25 0.00
Σf(A) = 542.00

If the length of the base is 150m, the common interval h = 150/10 = 15m (since
there are ten equally spaced stations).
15
Thus, the area = × 542 = 2710 m2
3

2.4. OTHER NUMERICAL INTEGRATION METHODS

Although Simpson’s rules are enough in most cases encountered in naval architecture,
there are a few other numerical integration methods which can be useful in certain
situations. We shall have a look at Tchebycheff’s rules here.

Tchebycheff’s two-ordinate rule

As with the Simpson’s rules, we start with a function which is expressed as a cubic
polynomial of x as in (2.1) for which the definite integral from x = –h to x = h is given
by (2.2).

We now assume that there is a value x = x1 and x2 = – x1 such that Area ABCD can be
expressed as M ( y1 + y 2 ) ,

where y1 = f ( x1 ) = a 0 + a1 x1 + a 2 x12 + a 3 x13 and


y 2 = f (x 2 ) = a 0 − a1 x1 + a 2 x12 − a 3 x13

Substituting these into the expression of area,

(
Area = M 2a 0 + 2a 2 x12 . )
Comparing this with (2.2) gives

M =h
1
Mx12 = h 3
3

h
Therefore, x1 = ±
3

28
In other words the area under the curve from x = –h to h can be obtained by
multiplying the sum of two ordinates at x = –0.57735h and x = 0.57735h with the
common interval h. This is for a special case where the base is divided equally about
the y-axis. In general, the first ordinate can be taken at x1 which is 0.42265h from the
left boundary of the base and the second ordinate similarly from the other end.

Tchebycheff’s three-ordinate rule

Again we start with (2.1) and (2.2), but this time we set up three ordinates at x = x1, 0
and –x, with their respective values of y1, y2 and y3. Again we express the area ABCD
as M ( y1 + y 2 + y 3 ) , and we know

y1 = f ( x1 ) = a 0 + a1 x1 + a 2 x12 + a 3 x13
y 2 = f (0) = a 0
y 3 = f (− x1 ) = a 0 − a1 x1 + a 2 x12 − a 3 x13

Substituting these into the expression for the area and comparing it with (2.2) gives

( )
Area = M 3a 0 + 2a 2 x12 = 2a 0 h +
2
3
a2 h 3
2 1
Therefore M = h and x1 = ± h
3 2

In other words the first and the third ordinates are at x = ±0.707107h . The area is
2h
then ( y1 + y 2 + y 3 ) .
3

We have seen two of Tchebycheff’s rules, but in fact there is a series of them for
various numbers of ordinates and corresponding order of polynomials. For us,
however, what we have examined here suffices.

2.5. POLAR INTEGRATION

We have discussed some integration methods with the Cartesian co-ordinate system as
the starting point for derivation. However, essentially all these numerical methods can
t2
be used for ∫t1
f (t ) dt . In some cases it is much easier to use polar co-ordinate system
and all these methods can, therefore, equally apply to polar integration. We shall first
have a look at how polar integration (sometimes known as radial integration) can be
used in certain situations.

Consider an area represented by two lines OA and OB and a curve segment AB as


shown in Fig.2.3. The angle of the line OA to the x-axis is θ1 and OB is θ 2 away from
the x-axis. We construct an elemental area dA of angle dα as shown at an angle α from
the x-axis. Since this area can be regarded as a triangle with the height of rdα and base
of r,

29
y

δα
r

α
θ2
x
O
θ1

Fig.2.3 Polar integration

1 1
dA = rrdα = r 2 dα
2 2

Therefore, the area is


θ2 1 2
A=∫ r dα
θ1 2

Also, the first moments of the elemental area about the x-axis and y-axis are
1 2 2 1
dM x = r dα ⋅ r sin α = r 3 sin αdα and
2 3 3

1 2 2 1
dM y = r dα ⋅ r cos α = r 3 cos αdα
2 3 3

Therefore,

1 3
θ2
Mx = ∫ r sin αdα
3 θ1

θ2 1
M y = ∫ r 3 cos α dα
θ1 3

You can work out how these integrals can be executed with the numerical integration
methods discussed in this Part.

This type of integration can be useful in the calculation of rotational stability, as will be
seen in the study of transverse stability.

30
NUMERICAL INTEGRATION TUORIAL

1. A curve has the following ordinates spaced 1.68m apart: 10.86, 13.53, 14.58,
15.05, 15.24, 15.28, 15.22. Calculate the area under this curve using
(a) Simpson’s first rule
(b) Simpson’s second rule
(c) the trapezoidal rule.

2. A curvilinear figure has the following ordinates at equidistant intervals:


12.4, 27.6, 43.8, 52.8, 44.7, 29.4 and 14.7.
Calculate the percentage difference from the area found by Simpson’s first rule
when finding the area by (a) Simpson’s second rule and (b) the trapezoidal rule.
Explain this discrepancy.

3. The effective girths of the outer bottom plating of a ship 27.5m between
perpendiculars are given below, together with the mean thickness of plating at
each ordinate. Calculate the volume of the plating. If the plating is steel of mass
density 7700 kg/m3, calculate the weight in meganewtons.

Ord No AP(0) 1 2 3 4 5 6 7 8 9 10(FP)


Girth (m) 14.4 22.8 29.4 34.2 37.0 37.4 36.8 28.6 24.2 2.6 23.2
t (mm) 10.2 10.4 10.6 11.4 13.6 13.6 12.8 10.4 10.1 10.1 14.2

4. The half-ordinates of a vessel 144 ft between perpendiculars are given below:

Ord No AP(0) 1 2 3 4 5 6 7 FP(8)


½ ord (ft) 17.0 20.8 22.4 22.6 21.6 18.6 12.8 5.6 0

In addition there is an appendage 21.6ft long abaft the AP whose half-ordinates at


equally spaced intervals are respectively, 0.0, 9.6, 14.0, 17.0 (AP).

Find the area and position of the centre of the area of the complete waterplane.

5. The cross sectional area of a sip 72m LBP, 11.5m beam and 4.3m draught are as
follows:

Station 0 ½ 1 1½ 2 3 4 5 6
CSA (m2) 0 17.1 28 35 40 45 46 46 46
Station 7 8 8½ 9 9¼ 9½ 9¾ 10
CSA (m2) 45 31 24 17 14.2 8 4.9 0

Determine the volume of displacement, mass of displacement in salt water, LCB,


CB, CP, and CM. Compute LCF and longitudinal second moment of the area.

31
PART 3 TRIM AND LONGITUDINAL STABILITY

Usual ships have sufficient longitudinal stability and, therefore, it is of little interest to us in
normal circumstances. However, it affects the trim of the vessel directly, and trim is a very
important factor in ensuring operational efficiency of ships. We shall briefly examine here
the longitudinal stability with emphasis on trim. There are three conditions of trim:

even or level keel


trim by the stern
trim by the head or bow

3.1 Longitudinal Centre of Floatation (LCF)

Consider the following figure which represents the forward wedge which submerges
when the vessel changes trim without alteration in its displacement.

For the elemental strip of half breadth y and length δx, the volume is 2yx tanθδx.
Therefore, the total volume of the wedge is

Lf
2 tan θ ∫ xydx.
0

Lf
But 2 ∫0
xydx is the first moment of the original waterplane about XOX’ .

0
Similarly for the aft portion of the vessel, the volume is − 2 tan θ ∫ − La
xydx.
(Note that La and Lf are length aft and forward respectively from XOX’.)

Since the vessel has not changed its displacement, the volumes of these two wedges
should be the same, i.e.

Lf 0 Lf
2 tan θ ∫ xydx = −2 tan θ ∫ xydx or ∫ xydx = 0
0 La − La

In other words, the first moments of the original waterplane (about the axis XOX’) fwd
and aft are the same in magnitude although opposite in sign. This shows that in order for
the vessel to change trim only, the trim axis XOX’ should be such that the first moment

32
of area of the original waterplane about this axis is zero. Put in another way, XOX’ must
pass through the centroid of the waterplane, and this point is called the centre of
floatation. The longitudinal position of CF is often referred to as LCF.

3.2 Moment to Change Trim

Consider the following figure which shows a vessel floating at waterline WL. A piece of
cargo of mass w tonnes is then moved horizontally through a distance h as shown. This
will cause the vessel to change trim about LCF and finally settle at waterline W1L1.
When the cargo is moved, the CG of the vessel moves from G to G1. After trimming the
CB will be vertically below G1 (since that is the new equilibrium).

The moment causing the trim is the same as the righting moment of the vessel at W1L1,
which is ∆GMLtanθ. If the change in trim is t and the length of the vessel is L, then
tanθ = t/L. We can see that the moment to alter trim by t is ∆GML t/L. Therefore, the
moment to change trim by 1cm (or MCT) is

∆GM L
MCT = tonne-m/cm.
100L

3.3 Changes in Loading Condition

Consider a vessel initially floating in equilibrium. Several masses (m1, m2, …, mn) are
loaded/discharged at distances (x1, x2, …, xn) from the original LCG. We wish to find the
final draft and trim when loading/discharging is complete.

A table is then prepared as follows:

MASSES LEVER ( from LCG) MOMENT


+ m1 + x1 + m1x1
- m2 + x2 - m2x2
- m3 - x3 + m3x3
+ m4 - x4 - m4x4
… … …
… … …
______ _______
Σm Σ mx

33
Note: + for masses added, fwd of LCG, trim by the bow
- for masses discharged, aft of LCG, trim by the stern.

(This sign convention is by no means universal!)

If the original displacement was ∆0,

∆ = ∆0 + Σm and

new LCG = (LCG)0 + Σmx/∆

LCB can be obtained from the table of hydrostatics at the displacement ∆. This will also
give the even keel draft and BML. (See an alternative approach of approximate
calculation below.)

Trimming moment = ∆ * (distance between LCB and LCG) and

Trim = (trimming moment)/(100*MCT) (metres).

3.4 Approximate Method of Calculating Trimming Moment

Consider a vessel originally floating at the waterline WL. At this condition the centres
of gravity and buoyancy of the vessel (G0 and B0) are vertically in line, since the vessel is
in equilibrium. Let xG the distance of G0 from the midship. (Note that the distance from
B0 to midship will also be xG.) A piece of cargo of mass m is loaded at a distance xm
from midship as shown in the diagram below.

Let the longitudinal distance of the new centre of gravity (G1) be xG’. Then

∆ ⋅ xG + m ⋅ xm
x G' =
∆+m

The parallel-immersed strip should displace water of mass m, and its centroid can be said
to be approximately at the LCF of WL. If the LCF is at distance xF from the midship,
then the new centre of buoyancy B1 will be away from the midship by xB, where

∆ ⋅ xG + m ⋅ xF
xB =
∆+m

As we have seen above, the trimming moment = ∆ * (distance between LCB and LCG).
The distance between LCB and LCG is

∆ ⋅ xG + m ⋅ xm − (∆ ⋅ xG + m ⋅ xF ) m( x m − x F )
x G' − x B = =
∆+m ∆+m

34
The trimming moment, therefore, is

m( x m − x F )
( ∆ + m) = m( x m − x F )
∆+m

This approximate method is useful when the approximation used in the derivation (the
centroid of the parallel-sinkage strip can be said to be at the LCF of WL) is valid. This
usually happens for a conventional ship-shape when the change in displacement is fairly
small (for example, draft change of a few cm rather than a few tens of cm). The method
allows the trimming moment to be calculated without having to find new LCG and LCB
first. With this method, the table given above will have levers calculated from LCF of
the original waterline and the moment sum is the trimming moment. This can then be
divided by the MCT for the new draft to produce the change in trim.

Example

A vessel 100m L x 30m B is operating at an even keel draft of 6m. The following
loading/unloading operations have been carried out at a port

Mass(tonnes) Loading/Unloading xg (m from midships)


100 L 10 fwd
50 U 40 aft
200 L 30 aft
50 U 20 fwd

Calculate the trim and consequently the drafts fwd and aft.

Solution

Mass Lever Moment


+ 100 + 10 + 1000
- 50 - 40 + 2000
+ 200 - 30 - 6000
- 50 + 20 - 1000
Σm = 200 Σmx = - 4000

∇ 0 = 100 × 30 × 6 = 18,000 m 3 ∆ 0 = 18000 × 1025


. = 18,450 tonnes
∆ 1 = 18,450 + 200 = 18,650 tonnes ∇ 1 = 18650 / 1025
. = 18,1951
. m3
. / (100 × 30) = 6.065 m, trimming moment = - 4000 tonne - m
T1 = 181951
I L = 30 × 100 3 / 12 = 2,500,000 m 4 BM L = I L / ∇ 1 = 137.4 m
MCT = 18,650 × 137.4 / (100 × 100) = 256.25 tonne - m / cm
trim = -4000 / 256.25 = 16 cm by aft.

Since LCF is amidships, trim forward and aft will be identical. Thus,

35
Taft = T1 + 016
. / 2 = 6145
. m, and T fwd = T1 − 016
. / 2 = 5,985 m

3.5 Derivation of BML

We have already discussed how the waterline appears to swing about the
transverse axis through LCF.

From the second figure of the notes, it can be seen that BB1 = BM L tan θ .

The centre of buoyancy moves to B1, because some volume has been transferred
from aft to forward. Referring to the first diagram in the notes, the longitudinal
moment of volume transfer can be calculated by
Lf Lf Lf

moment = ∫ ( elementary volme ) × x = ∫ ( 2 yx tan θ dx ) × x = 2 tan θ ∫


− La − La − La
yx 2 dx

The last item in the above expression is actually the longitudinal second moment
of the waterplane, IL, multiplied by tanθ. Also the shift in centre of buoyancy,
BB1, can be obtained by dividing the moment by the total volume.

Therefore,

BB1 = I L tan θ ∇ = BM L tan θ


∴ BM L = I L ∇

As stated earlier, some people prefer to use BML instead of GML, but, wherever
possible, GML should be used.

36
Part 4 STATICAL TRANSVERSE STABILITY

4.1 INTRODUCTION

All ships should have adequate stability in all their operating conditions. This statement
sounds almost too obvious and simple to deserve a serious consideration, but it raises
two important questions: firstly, what is stability; and secondly how do we define
‘adequate’? We cannot even begin to answer the second until we have answered the
former.

The word ‘stability’ can mean different things to different people: for example,
mathematicians talk about the stability of solutions, dynamicists discuss the stability of a
system, and chemists worry about the stability of a compound they are working on, while
many people know roughly what is meant by economic, political or social stability.
Although stability is used in Naval Architecture in a variety of contexts as well, for
example in ‘directional stability’, the unqualified word ‘stability’ in normal usage has a
specific meaning, reflecting our preoccupation in keeping a ship upright, implying that
achieving this aim cannot be taken for granted.

A more precise definition of ship stability may be ‘the ability of the ship to return to the
normal upright condition, when disturbed from this attitude, without endangering itself
or the cargo and human life it carries’. The keyword here is ‘return’, and this means that
the disturbance from the normal upright condition (equilibrium), whatever caused it, will
automatically generate an opposing force or moment to return it to the equilibrium. The
nature of these ‘restoring’ forces and moments will become clearer if we examine the
cases of disturbances in different directions.

First of all, it is easy to see that the disturbances on the horizontal plane, i.e.
displacements along x- and y-axes and rotation about z-axis (surge, sway and yaw
respectively) will not generate any restoring force or moment, as the system in these
cases is in complete neutral equilibrium. Displacement along z-axis (heave) can be
relied on to produce restoring force, provided the waterplane area is non-zero. In the
case of normal ships this, therefore, is no problem, except that there should be sufficient
free board so that the ship faces less chance of being swamped. In any case, the
disturbance in heave direction is straight forward and does not require complicated
theory to explain the phenomenon.

On the other hand rotational disturbances about x- and y-axes (roll and pitch, or their
static equivalent of heel and trim) are very different indeed. One knows, instinctively,
that the disturbance about the y-axis (i.e. trim) is no problem, because longitudinal
stability is so high for conventional ships. Nevertheless this is an important factor in
determining the performance of ship propulsion and has been discussed in Part 3.

This leaves the stability in the transverse direction as the really important issue as we
cannot be sure that the ship will generate sufficient moment to return to upright condition
when heeled and how much heel the ship can suffer without completely capsizing.

We shall, therefore, examine the transverse stability carefully, with particular


emphasis for the cases where the watertight integrity of the ship is intact (intact
stability). When we are completely at ease with this we shall examine the
approaches normally used for studying the stability when the ship is damaged and
the normally watertight skin of the ship is broken.

37
4.2 BASIC PRINCIPLES OF STABILITY

Before we start examining the nature of the restoring moment (or more
commonly known as ‘righting moment’), we need to make some fundamental
assumptions as follows:

(a) the ship is floating in calm water


(b) the ship is stationary
(c) the ship is rigid, i.e. it maintains its original shape whatever it might assume.

Some of these assumptions can be removed either partially or wholly, and we


shall deal with that later.

Righting Moment

We shall now have a look at the mechanism through which the righting moment
is generated when an arbitrary object is inclined from an equilibrium position.
Consider an object floating in water at equilibrium. The weight of the object is W
and its centroid of mass is at G. Since the object is in equilibrium the underwater
W
volume of the object will be , where ρ is the mass density of the water and g
ρg
is the gravitational acceleration. Furthermore the zero sum of moments dictates
that the centroid of the underwater volume, B, be vertically under G as shown in
Fig.4.1(a).

L1
G G
W L
φ
W1 B1
B

Fig.4.1(a) Object in equilibrium Fig.4.1(b) After


heeling

If this object is then inclined by an angle φ , as shown in Fig.4.1(b), without


altering the displacement, the centroid of the underwater volume will move to a
new location B1, while the mass centroid of the object remains at G. It is easy to
see that except in very special circumstances, B1 will in general move out of the
vertical line passing through G. The sum of the forces is zero but a moment is
created. Therefore, the object is no longer in an equilibrium, and the resulting
moment may be in either direction. We shall define this moment as the ‘righting
moment’, and it will be positive if it tends to put the object back to the original
upright position, and negative if it tends to increase the angle of inclination. The

38
magnitude of this moment is simply the product of the weight W and the distance
between the two vertical lines through G and B1.

Note that a ‘heeling moment’ tends to increase the angle of inclination.


Therefore, a positive heeling moment will be in the same direction as a negative
righting moment. If we divide the righting moment by the weight of the object,
we obtain ‘righting lever’ or ‘righting arm’. Of course we can define ‘heeling
lever’ and ‘heeling arm’ in a similar manner.

Symbols and Terminology

When stability is discussed in naval architecture we need to define the meaning


of certain terminology and symbols. While some authors use their own systems,
the most commonly used symbols and terminology are illustrated in Fig.4.2.

L1

Z
W G L
O φ
W1 B1
B

N
K

WL: upright waterline


W1L1: heeled waterline
G: centre of gravity of the vessel
B: centre of buoyancy (upright)
B1: centre of buoyancy (heeled)
K: keel (or the point at which the baseline crosses the centreline)
M: metacentre
φ: angle of heel
KB: height of V.C.B.
KG: height of V.C.G.
GM: metacentric height
BM: metacentric radius
GZ: righting lever (or righting arm)

Fig.4.2 Definition of terms

39
Metacentric Height

It is convenient to assign a positive or negative sign to the angle of heel, and


throughout this course we shall use the convention that heel to starboard is
positive and heel to port is negative (viewed from the stern). For example, a heel
angle of 10° means 10° heel to starboard and -10° denotes 10° to port.

A vessel floating in equilibrium at WL is heeled to starboard by an angle φ so that


the new waterline can be represented by W1L1. Note that at this waterline the
displacement should be the same as the original one, and for a small angle of heel
W1L1 can be assumed to pass through point O of Fig.4.2. The CG of the vessel is
still at G (assumed to be on the centreline of the vessel), while CB will move to
B1. A righting moment is generated as indicated by the two forces equal in
magnitude, but opposite in direction acting on two vertical lines separated by a
distance GZ. Thus,

M R = GZ × ∆

GZ is the righting lever.

For small φ, it can also be shown that


GZ = GM sin φ ≈ GMφ

Remember that φ is in radians in the above expression.

Now that we have the stability quantified, albeit in a very rudimentary way, we
need to examine what the point M signifies and how we can calculate GM. Point
M, or the metacentre, is formally defined as the point of intersection of the two
vertical lines passing through the centres of buoyancy at two very close angles of
inclination. It is easy from this definition to see why the length BM is called
‘metacentric radius’. The metacentre at the heel angle of zero is called the
initial metacentre, or more often simply as the metacentre. The metacentres at
non-zero angles of heel are known as pro-metacentres. It is easy to observe that

GM = BM + KB − KG

Therefore, if we can calculate BM and KB, and if we know KG, we can obtain
GM. The method of calculating KB and KG has already been discussed in Part 1,
and thus the problem is reduced to determining BM.

Consider Fig.4.3, which shows the whole ship and we attempt now to calculate
GM (called metacentric height, or more correctly initial metacentric height).
The displacement in the heeled state is the same as the initial displacement.
Therefore, the volume of the emerged wedge (‘out’ wedge), v1, should be the
same as that of the immersed wedge (‘in’ wedge), v2. In other words, v1 = v 2 = v ,
say.

40
dx

L/2

L/2 M
dx

L1

2y/3 2y/3
G φ
W g2 L
g1

W1 B B
y

K
'in' wedge
'out' wedge

Fig.4.3 Derivation of GM

2
For very small angles of heel, the local centre of gravity of the wedges is y
3
away from the centreline in transverse direction. The moment of transfer of
buoyancy for this thin strip of thickness dx is

2 ⎛1 ⎞ 2
2× y⎜ y ⋅ y tan φ ⎟dx = y 3 tan φdx
3 ⎝2 ⎠ 3

If we now integrate this moment along the whole length of the ship (i.e., from
L L
x = − to x = ), we obtain the total moment of transfer for the whole ship.
2 2
L L
2 3 2
∫−
2
L
3
y tan φdx = tan φ ∫ 2L y 3 dx
3 −
2 2

We can relate this moment to the transverse movement of the centre of buoyancy
which we will call h, as shown in the enlarged figure below.

From the moment theory we know that

41
L
2
tan φ ∫ 2L y 3 dx
3 −
h= 2

On the other hand, for small φ

h ≈ BM tan φ
Therefore
L
2 2 3
3 ∫− 2
BM = L y dx / ∇

Note that y is half breadth of the waterline, and therefore is a function of x.


L
2
The integral ∫ 2L y 3 dx happens to be the second moment of the waterplane area
3 −2
about the centreline, or IT (transverse second moment of waterplane).

Implications of GM

Having found a way of calculating GM, we now need to explore its implications
a little before we go further into stability.

(i) GM as a stability parameter

As we have seen, the righting moment of a vessel when heeled to a small


angle φ is

Righting moment = ∆GM sin φ ≈ ∆GMφ

In other words GM is a direct measure of stability at small angles of heel.


The reason why people prefer GM as a parameter to GZ, for example, is
that GM does not vary whilst GZ is a function of φ . Thus, GM is a
parameter showing the initial stability characteristics of the vessel.

For reasons which will become apparent later on, at larger angles GM often
gives misleading information about stability and consequently it is unwise
to use it alone to indicate stability.

(ii) GM as a roll motion parameter

Although the full implication of GM in terms of roll motion characteristics


have to be explored later, it is worth noting at this stage that the natural
period of roll motion of a vessel is a function of GM, i.e.

2πk
Tn =
GM ⋅ g

42
where

Tn = natural roll period


k = radius of gyration (often about 0.42 B)
g = gravitational acceleration

It can be seen, therefore, that a vessel with high initial GM (i.e. a ‘stiff’
ship) will have short natural roll period, and a ‘soft’ ship (low GM) will
have long natural period. A stiff vessel will roll relatively rapidly in a jerky
fashion which may be detrimental to the comfort of passengers and crew.
Note the use of terms analogous to the characteristics of a spring.

(iii) Methods of influencing GM

The easiest way of changing GM of an existing vessel is by using ballast,


often in solid form. This often is an expensive, albeit necessary, way of
increasing GM, as it reduces the carrying capacity of the ship. At the
design stage, however, any perceived shortcomings in intact transverse
stability can be remedied by increasing the second moment of the area,
either by increasing the breadth of the ship, where it is possible, or
changing the shape of the waterplane. It must be remembered that the
waterplane is often governed by the requirements of speed or resistance.

4.3 FACTORS AFFECTING GM

The GM calculated as above may not be the effective GM for the ship depending
on its conditions. There are a number of factors which need to be taken into
account when trying to arrive at an effective GM and the two typical cases are
suspended weight and free surface effects.

Suspended Weight

Consider a vessel with a weight of mass m suspended from point P as shown in


Fig.4.4(a). First, if we assume that the weight is tied to the post so that it does not
move, then the righting moment of the vessel for a small angle of inclination φ is

RM = ∆ ⋅ GM sin φ

(Note that the righting moment in this expression is mass moment, and not force
moment. This is the usual convention but there are occasions when one needs to
use force moment. So watch out.)

43
P

Fig.4.4(a) Suspended weight in upright condition Fig.4.4(b) Suspended


weight heeled

If the weight is gently released in this heeled condition, the weight will move to a
position vertically below the point P as shown in Fig.4.4(b).

Then the movement of the c.g. of the weight is

gg1 = h sin φ

we can now approach the problem in two ways.

(i) The heeling moment due to the shift of the weight c.g. is m ⋅ h ⋅ sin φ
Therefore the net righting moment =
(∆GM − mh ) sin φ = ∆⎛⎜ GM − mh ⎞⎟ sin φ
⎝ ∆ ⎠

In other words, GM is effectively reduced by mh / ∆ .

(ii) The shif of the vessel C.G. due to the shift of the weight c.g. is

mh sin φ
GG1 =

Therefore, the net righting moment


⎛ mh ⎞
= ∆ (GM sin φ − GG1 cos φ ) ≈ ∆⎜ GM − ⎟ sin φ ,
⎝ ∆ ⎠
since cos φ ≈ 1 for φ << 1.

44
This is the same expression as obtained in (i).

mh
The apparent reduction in GM of is equivalent to an increase in VCG by the

same amount and this signifies that, when evaluating the VCG of a vessel, any
suspended weight free to swing should be considered to be situated at the points
of suspension.

Free Surface

The free surface effects are by far the most important factor which influences the
magnitude of initial GM. These effects occur when there is any container with
liquid content having free surface, i.e. in the case of an enclosed tank the tank is
partially filled so that the surface of the liquid is free to maintain horizontal
position whatever the ship’s attitude (termed a ‘slack’ tank).

Consider a vessel with a tank partially filled with liquid of density ρ T . The
density of the water the ship is floating on is ρ W .

The righting moment of the vessel if the surface is not allowed to move (e.g. by
assuming a rigid surface film) at a small angle of heel φ is ∆ ⋅ GM sin φ . This
righting moment is often called ‘solid’ righting moment, and GM is called solid
GM and sometimes denoted as GMs.

W φ L

g g1

2b

Fig.4.5 Vessel with a ‘slack’ tank

Now the thin film on the surface is removed and the liquid is allowed to move.
For the sake of clarity we shall consider a rectangular tank of breadth 2b and
length l as shown in Fig4.5. The point m denotes the intersection of the vertical

45
lines through g and g1, where g is the original centre of gravity and g1 is the
centre of gravity of the liquid at the heeled condition. Then

2 2
The moment of volume transfer = b ⋅ b ⋅ b tan φ ⋅ l = b3l tan φ
3 3
Therefore,
2 3 i
gg1 = gm tan φ = lb tan φ v = T tan φ .
3 v

Again there are two approaches.

(i) The c.g. of the liquid will move from g to g1 and therefore the C.G. of the
vessel moves to G1.

w ⋅ gg 1 w ⋅ gm tan φ w iT i vρ T i ρ
GG1 = = = tan φ = T tan φ = T T tan φ
∆ ∆ ∆ v ∆ v ∆
where
w is the liquid mass = vρ T
v is the liquid volume
iT is the local transverse second moment of the free surface area.

Thus, the net righting moment

⎛ i ρ ⎞
= ∆(GZ − GG1 cos φ ) = ∆⎜ GM − T T ⎟ sin φ = ∆(GM − GG2 ) sin φ ,
⎝ ∆ ⎠
say.

iT ρ T
This item GG2, which is equal to , is called the ‘free surface

iT ρT
correction’. The term GM − is known as the GM fluid and denoted

GMf.

It can be seen that the effect of the free surface is to increase the VCG
effectively by this amount. In other words, recalling a similar situation with
suspended weights, the c.g. of the liquid appears to be at the point m and not
at g.

(ii) Heeling moment due to the shift of the liquid c.g. is

w ⋅ gg1 cos φ = w ⋅ gm tan φ cos φ = w ⋅ gm sin φ

Therefore, the net righting moment is

46
⎛ w ⋅ gm ⎞
∆GM sin φ − w ⋅ gm sin φ = ∆⎜ GM − ⎟ sin φ
⎝ ∆ ⎠
⎛ i ρ ⎞
= ∆⎜ GM − T T ⎟ sin φ
⎝ ∆ ⎠

as obtained in (i).

It is not difficult to see a certain similarity between free surface effect and
suspended weight effect.
For suspended weight
⎛ mh ⎞
net righting moment = ∆ ⎜ GM − ⎟ sin φ
⎝ ∆ ⎠
For free surface
⎛ w ⋅ gm ⎞
net righting moment = ∆ ⎜ GM − ⎟ sin φ
⎝ ∆ ⎠

4.4 INCLNING EXPERIMENT

It can be seen from the above that in order to get an accurate idea of stability we
need to know the accurate position of the VCG of the vessel. The VCG is
estimated at the design stage, but this estimation is not sufficient. Further more,
the VCG can change after being in service for a while. Therefore, all ships have
to undergo a measurement process known as inclining experiment or test after
launch and at regular intervals thereafter.

Since directly measuring KG is not possible, GM is estimated by imposing a


known heeling moment and measuring the resulting heel angle. Since KM can be
calculated from the hydrostatics, KG can be estimated from this.

A clear windless day and a site with minimum current is selected for this
experiment. All mooring lines are slackened off and all non-essential personnel
are taken off the ship. There should be no suspended weight or slack tank, but, if
it is not possible to ensure this, the details of these should be taken so that they
can be taken into account. The draught marks around the ship are read off so that
the displacement, VCB and LCB can be calculated. Any items which are not a
normal part of the ship should also be removed or if it is not possible taken a
careful note of.

A number of similar weights, w, are then moved across the deck through a known
distance, h, so as to incline the vessel to both port and starboard by a small angle.
The angles of inclination are carefully measured at each stage by either an
inclinometer or a long pendulum with a plumb bob in a bucket of oil to damp its
movement. If identical inclining weights are used, then an average change in
inclination after each movement of the weight is obtained, and then

47
wh
GG1 =

but GG1 = GM T tan φ
wh
Therefore GM T =
∆ tan φ

If a pendulum is used, tan φ can be obtained by dividing the horizontal


movement of the pendulum by its length. Having obtained KG
( KG = KB + BM − GM T ), it is relatively simple to undertake corrections for
removing non-lightship items, as we need to keep a careful track of KG for the
light ship condition.

4.5 LARGE ANGLE STABILITY

The stability characteristics of a vessel at large angles of heel are in general very much
different from those of initial stability or stability at very small angles of heel. It may be
recalled that stability of a vessel can be quantified in terms of righting moment or, for a
given displacement, righting arm. Moreover, the righting arm could be represented by
using the initial metacentric height in the form

GZ = GM sin φ

The most essential assumption in the derivation of this expression was that φ << 1. In
other words, at very small angles of heel we can take the initial metacentric height as the
stability parameter.

However, as φ increases, this relationship becomes less valid and beyond the heel angle
of about 10° GM loses its meaning as the stability parameter entirely. Therefore, it is
now necessary to go back to the righting arm for a meaningful indication of stability.
This section examines various ways of calculating the righting levers at large angles of
heel, but first we shall discuss what happens to the by now familiar terms of metacentres
etc at large angles.

Metacentres at Large Angles

When dealing with initial stability, BM =


IT was developed with the assumption that

the centre of buoyancy moves horizontally, i.e. that the transfer of wedges was
horizontal. Obviously this cannot be true at large angles and the centre of buoyancy
moves out round a curve called the isoval or metacentric involute.

48
When the ship is upright, this curve is perpendicular to the centreline of the ship and the radius of
curvature ρ is B0 M = I T / ∇ . The verticals through the heeled centres of buoyancy do not
normally intersect at M and any two verticals very small angles apart will have a corresponding
metacentre - the pro-metacentre for that angle. If a curve is drawn from M through these pro-
metacentres, this is the metacentric evolute.

Attwood's Formula (1796)

Consider a vessel heeled to a large angle φ without any change of displacement. Then the volume of
the immersed wedge LSL1 must equal that of the emerged wedge WSW1, but the inclined waterline
WL will not necessarily pass through the point O. Let the volume of the wedges be v with centroids
g1 and g2, and the volume displacement be ∇ .

G Z v
g1
W1 h2 L1
h1 g2 φ
L
v B R
B1

Fig.4.6 Attwood’s formula

Horizontal moment of buoyancy transfer = v ⋅ h1 h2 = ∇BR

Therefore, ∇BR = v ⋅ h1 h2

Also from the figure BR = GZ + BG sin φ

Therefore, GZ = v ⋅ h1 h2 / ∇ − BG sin φ

This is known as Attwood's formula, but in a way it is incomplete, because it does not say how to
obtain v ⋅ h1 h2 . This is done by Barnes for example (see below).

49
Barnes's Method

'
L1
h2
h2 L2
W F2 b2 '
L
O b2
b1 b'1

h1 h' 1
W1
W2

Fig.4.7 Barnes’s method

W2L2 is the waterline we are looking for.


v1 = buoyancy of emerged wedge between WL and W1L1
v2 = buoyancy of immersed wedge between WL and W1L1
v = buoyancy of emerged and immersed wedges between WL and W2L2
buoyancy of the layer between W1L1 and W2L2 = v2 – v1
b1' , b2' : centroids of v on each side
b1 , b2 : centroids of v1 and v2 respectively
h1 , h2 : projections of b1 and b2 onto W1L1 respectively
h1' , h2' : projections of b1' and b2' onto W1L1 respectively
F2 is the projection of centroid of the layer (v2 – v1) onto W1L1, but since the layer is very small,
we can assume it to be identical to the t.c.f. of W1L1.

The moment of volume transfer is

v ⋅ h1' h2' = v1 ⋅ Oh1 + v2 ⋅ Oh2 − ( v2 − v1 ) OF2

If we evaluate the RHS of this relation, then we can use Attwood's formula. By using radial
integration and introducing a dummy variable α as defined in the Fig.4.8,


⎛1 2r 1 2r ⎞
v1 ⋅ Oh1 + v2 ⋅ Oh2 = ∫ ∫ ⎜ r12 dα 1 + r22 dα 2 ⎟ cos (φ − α ) dx
0 0⎝
2 3 2 3 ⎠

⎛ r 3 + r23 ⎞
= ∫ ∫⎜ 1 ⎟ cos (φ − α ) dα ⋅ dx
0 0⎝
3 ⎠

where r1 and r2 are distance between the point O and side hull port and starboard respectively.

v1 and v2 can be calculated in a similar manner as follows:


1
v1 = ∫ ∫ r12 dα ⋅ dx
0 0
2

50

1
v2 = ∫ ∫ r22 dα ⋅ dx
0 0
2

L
Waterplane area = ∫ ( rφ
0
1 + rφ 2 )dx

where rφ 1 and rφ 2 are r1 and r2 at α = φ respectively.

First moment of area about the longitudinal axis through O is

L
⎛ 1 1 ⎞
∫ ⎜⎝ − 2 rφ
0
1 + rφ 2 ⎟dx
2 ⎠

Then
OF2 = (first moment)/(area) (note: positive to sta'b'd)

This method can produce the righting arm value at any heeling angle without having to find the true
heeled waterline W2L2 first.

L1

δα L
O
α

Fig.4.8 Radial integration used for stability calculation

Wall-Sided Formula (Scribanti)

If the vessel is wall-sided at all points in the length, irrespective of the waterline shape, the areas of
immersion and emersion at each section will be equal and opposite right angled triangles and the
heeled waterlines for the same displacement will all intersect at the same point on the centreline.

Then by geometry
2
distance between g1 and g2 perpendicular to WL = y tanφ
3
area of each triangle = ½ y2 tanφ
4
distance between g1 and g2 parallel to WL = 3y

Therefore, moment of transfer of buoyancy parallel to WL


L
1 4
∫ (2 y tan φ · y )dx
2
=
0
3

51
L
2
= tan φ ∫ y 3 dx
3 0

Moment of transfer of buoyancy


perpendicular to WL
L
⎛1 2 ⎞
∫ ⎜⎝ 2 y tan φ ⋅ y tan φ ⎟dx
2
=
0
3 ⎠
L
1 2
= tan φ ∫ y dx
3

3 0

L
2 3
But ∫ y dx = IT , i.e. the transverse moment of inertia of the waterplane.
30

Therefore
∇X = IT tan φ or X = BM tan φ
1 1
∇Y = IT tan 2 φ or Y = BM tan 2 φ
2 2

If the transfer of buoyancy had been purely horizontal (i.e. from B to P), the righting lever would
have been

GH = GQ sin φ

But X = BQ tan φ and also BM tan φ . Therefore, Q is the initial metacentre M.

y y Q
L1 (M)
Z
φ
ytan φ H
W φ g2
L
G
ytan φ g1
W1
B1
B1
Y
B Y
X B X P

Fig.4.9(a) Wall-sided formula Fig.4.9(b) Geometry of GZ in wall-sided formula

52
The righting lever then is
GZ = GH + HZ = GM sin φ + Y sin φ
⎛ 1 ⎞
= ⎜ GM + BM tan 2 φ ⎟ sin φ
⎝ 2 ⎠

This formula gives a close approximation of GZ for ships with normal form below the
angle of deck edge immersion.

Example

Consider a log of a constant square section of a homogeneous material with the


specific gravity of 0.5 floating in fresh water. Its length is L and the side of the
square is a.

a 1
V = L⋅a⋅ = L ⋅ a2
2 2
1
I= L ⋅ a3
12
2L ⋅ a3 a
BM = =
12 L ⋅ a 2 6
a
KB =
4
a
GM = KB + BM − KG = −
12

I.e. the log is unstable when floating on one of its sides. The equilibrium point is
when
GZ = 0. Based on the wall-sided formula,

−2GM 2a 12
tan φE = = =1
BM a6
∴φE = 45°

In other words the log floats with a corner down.

53
4.6 CURVE OF STATICAL STABILITY

We can learn a great deal about the stability of the vessel by looking at its intact still
water stability curve, and here we shall discuss some of the key features of stability
curves which give us the clue.

A typical stability curve is shown in Fig.4.10.


Righting moment

Angle of heel

Fig.4.10 A typical intact stability curve

(a) At small angles of φ, GZ = GM sin φ . Therefore,

dGZ dφ = GM cos φ = GM at φ = 0
dGZ dφ is the slope of the stability curve and it can be concluded therefore that
the slope of the righting lever curve at φ = 0 is GM. See Fig.4.11.

GZ

GZ max

GM

1 rad φ
(57.3°)

54
Fig.4.11 GM and maximum GZ on the curve

(b) ∆GZ max is the maximum heeling moment the vessel can withstand without
capsizing. See Fig.4.11.

(c) If C.G. of the vessel is not on the centreline, as static heel can occur.

(d) If C.G. moves say from G to G1 vertically, the new righting lever at heel angle φ is

G1Z1 = GZ − GA = GZ − GG1 sin φ for all angles. See Fig.4.12.

(e) If C.G. moves transversely from G to G1, the new righting lever at angle φ is

G1Z1 = GZ − GG1 cos φ for all angles. See Fig.4.13.

L1 L1
G1 Z1
W φ L W φ L
Z Z
G G
W1
A W1 G1 Z 1

B B1 B B1

Fig.4.12 Vertical movement of C.G. Fig.4.13 Horizontal movement of


C.G.

(f) The area under the righting moment curve represents work or ability of the vessel to
absorb energy imparted to it by external forces. Work required to heel from φ2 to φ1
φ2
is ∫ ∆ ⋅ g ⋅ GZ ⋅ dφ .
φ1
See Fig.4.14 (in this diagram the term ∆ ⋅ g is omitted, as both

items are constant). If the vessel moves from angle φ2 to φ1, the same amount of
work will be done on the vessel in the form of increased kinetic energy in the form
of increased roll velocity. It is also important to note that indeed the area under the
curve is a kind of potential energy (remember different forms of energy can be
mutually converted).

55
GZ

φ1 φ2 φ

Fig.4.14 Work done to change heel from φ1 to φ2

(g) ‘Range of (positive) stability’ is the range of angles within which the vessel has
positive stability. The final point at which the stability becomes negative is known
as the ‘vanishing point’. See Fig.4.15.

GZ GZ

vanishing angle

vanishing angle

φ φ
range of stability range of stability

Fig.4.15 Illustration of vanishing angle (φv) and range of stability

(h) Some curves have concave and some others convex shape at the beginning. Think
about which one is preferable for the same GM.

(i) A negative GM does not necessarily mean the vessel will capsize. It may have a
range of positive stability. Note here that the point of positive equilibrium is at the
point where the curve crosses the angle-axis upwards for positive angles.

(j) For a symmetric vessel with the C.G. at the centreline, the curve can be expressed as
an odd function of heel angle, for example, GZ (φ ) = a1φ + a3φ + a5φ + ...
3 5

4.7 ISOCLINE METHOD

We have seen that one of major concerns of the foregoing methods is that the
displacement at every heel angle should be identical to the one of upright position. This

56
approach may be called constant displacement method. Clearly the major issue here is
having to find the correct heeled waterline (even though Barnes's method gets around
this problem by approximation). There is another approach which makes no attempt to
keep the displacement constant whatever, and it is called isocline method.

The isocline method is an indirect way of obtaining a statical stability curve, and indeed
it has an additional advantage that it can cater for any alteration in the loading condition.
The result of this method when presented in a graphical form is called cross curves of
stability, an example of which is shown in Fig.4.16. It is a convenient and effective
method for normal ships but may not be as effective for vessels such as semi-
submersibles because the cross curves will then have many discontinuities. A
conventional hull form may require only about 10 or so drafts and the results are then
faired into smooth curves.

The essential procedure of this method can be summarised as follows:

(a) Decide on the angles of heel (often these are 15, 30, 45, 60, 75 and 90°).
(b) For each angle of heel draw several waterlines at least covering the range of
displacement of interest.
(c) Compute the displacement and righting moment about the vertical line through an
assumed centre of gravity (assumed KG is necessary because KG will change
according to the loading condition and hence displacement).
(d) Plot the righting moments or righting levers on the base of displacement, one curve
for each heel angle.

The resulting curves can also be presented in the form of solid of stability which is in
fact cross curves drawn upon two bases, viz displacement and angle of heel, forming a
surface.

Often KG of 0 m is assumed and in this case the cross curves are sometimes called KN
curves. Whatever value of KG is assumed, it is essential that it is clearly shown in the
curves. When using the curves to generate a statical stability curve for a given
displacement, it is then necessary to be corrected for the actual KG for that loading
condition. This correction is called sine correction (see 4.6 (d) above), since it is of the
form

GZ = ( GZ )0 − G0G sin φ
where (GZ)o = GZ values obtained from the cross curves
Go is the assumed centre of gravity
GoG is distance between the assumed and real centre of gravity (positive if G
is above Go and negative if below).

57
GZ CROSS CURVES OF STABILITY
(m) 30° Assumed KG = 3m
45°
3.5
60°
75°

3.0 15°
90°
90° 75°
2.5 60°
10°

45°
2.0

30°

1.5

1.0
15°

10°

0.5

1000 2000 3000 4000 5000 6000 7000 8000 9000 10,000 11,000
Displacement (tonnes)
Fig.4.16 An example of cross curves of stability

58
STATICAL STABILITY TUTORIAL 1

1. A box-shaped barge of 100mL x 10mB x 3mT is floating in calm sea.

(a) Assume a heel angle of 5° and plot the section on a graph paper. Find the new
centre of buoyancy and graphically locate the metacentre. Compare the BMT thus
obtained with calculated value. Comment on the result.

(b) If the VCG of the barge is 4m above the baseline, find the initial metacentric height.

(c) How high the centre of gravity can be raised without causing GM to become
negative.

(d) With KG = 4m, increase length, breadth and draught by 10% one at a time and
compute GM for each case. Comment on the results.

2. A log of square section with each side 20cm long is floating in calm fresh water. The
specific gravity of the log is 0.5 and the log is homogeneous.

(a) Find the initial GM

(b) Find the attitude of stable equilibrium.

3. A box-like vessel of 100mL x 20mB x 10mD is operating in sea water at a draught of 6m.
In this condition KG = 7m. It has a box-shaped oil tank measuring 10mL x 10mB x 3mD.
The centreline of the tank is at midship and the tank is initially full. Assume that the
tank rests on the bottom of the vessel.

(a) Find the initial GM.

(b) If half of the oil (specific gravity 0.9) is pumped out, what is GMf.

(c) Instead of pumping the oil out as in (b), one third of the original full tank content is
pumped into a settling tank measuring 10mL x 5mB x 3mD and situated right above
the oil tank. Find GMf for this situation.

4. A box-like vessel, 180mL x 31.5mB, floats in sea water at a level keel draught of 12m and
in this condition the value of GM is 0.891m. A ‘tween deck space, 30mL x 31.5mB, is
accidentally flooded with sea water to a depth of 1.35m. The deck on which the water is
lying is level and is 12.6m above the baseline of the vessel. Estimate the virtual GM of
the vessel.
(Note: GMf is sometimes called virtual GM.)

5. A vessel 100m long and 6m deep is of a constant rectangular section, 12m broad and
symmetrical about the centreline. It is floating in sea water initially at a draught of 3m
even keel. Weight is added so as to trim the vessel in such a way that the deck at one
end is just immersed while the draught at the other end remains at 3m. Calculate the
magnitude of the weight required and the position of its centre of gravity.

6. A rectoidal vessel, 50mL x 12mB, is operating in sea water at a level keel draught of 3m.
It goes aground on a narrow ledge 15m from the aft end on the centreline of the vessel.
The tide then falls by 0.5m. Find its final attitude.

59
STATICAL STABILITY TUTORIAL 2

1. A box-shaped vessel, 36m long x 6.4m beam, floats in sea water at a level keel draught
of 2.44m. In this condition the value of GM is 0.789m.

Cargo is now loaded, part with c.g. at 1.22m above the bottom of keel and the remainder
with c.g. at 2.5m. The final draught is 2.75m even keel and the final KG is 1.804m.
Find the weight of cargo loaded at each position.

2. The following data, which may be assumed to be constant except for Taft, are given for a
vessel as follows:

TPC = 18.0 tonnes/cm, MTC = 160 tonne-m/cm


LCF is amidships, original Taft = 10.0m

After ballast is discharged from a tank with c.g. at 52m aft of midship, the vessel is
found to float at a mean draught of 9.5m and to have a trim of 50cm by the stern.
Calculate the amount of ballast discharged and the original draught forward.

3. A rectangular vessel 100mL x 20mB x 8mT goes aground at one end. VCG of the vessel
is 10m and LCG is amidships. The tide then falls by 1m. In order to effect refloating of
the vessel several pieces of cargo weighing total 1000 tonnes with their centroid at 30m
aft of midship are to be moved. Find the minimum horizontal distance by which they
have to be moved.

4. A box-like barge, 80mL x 15mB x 10mD, is operating in sea water at an even keel draught
of 3m. When some cargo of mass 200tonnes was moved horizontally in the transverse
direction by 3.2m, the barge heeled by 3°.

- Estimate GMT and BMT, and hence the position of the VCG.
- If the same cargo is moved aft by 10m instead of transversely, what will be the trim
and hence the draughts forward and aft?
- Having returned the moved cargo to its original position, it is now decided to load the
barge with some containerised cargo of mass 2000tonnes on the deck. The centre of
gravity of the additional cargo when loaded will be at 11.5m above keel. Estimate the
transverse metacentric height.
- What is the allowable cargo mass which can be loaded on the deck, if the minimum
allowable GMT is 1m? (Ignore trim and assume the c.g. of the cargo at 11.5m above
keel.)

5. A semi-submersible comprises two rectangular cross-section pontoons, each 90m long,


11m wide and 7m deep. The deck is supported by four cylindrical columns each 9.5m in
diameter, the centrelines being spaced 70m apart longitudinally and 60m apart
transversely. The vessel floats at a draught of 23m in sea water and has a vertical centre
of gravity 18.5m above the keel. Calculate the transverse metacentric height.

A mass of 800tonnes is added on board at a position 35m above the keel. Calculate the
new values of draught and vertical centre of gravity.

60
PART 5 DAMAGE STABILITY AND WATERTIGHT SUBDIVISION

5.1 INTRODUCTION

A damage to a vessel which compromises the watertight integrity of the hull will lead to
ingress of water into the compartment(s) of the vessel. The flooding consequent to the
broaching of the watertight skin of the ship will affect the attitude of the vessel, i.e.
trim, draught and heel, and the stability characteristics will be affected, usually to the
worse. The ‘remaining’ stability after sustaining damage is known as residual or
damage stability.

All types of vessels are subject to risk of being lost if they are damaged whether by
collision, grounding or internal mishaps such as fire and explosion. Such accidents are
frequent enough in practice that some degree of protection against the eventualities of
flooding should be given. For example, sufficient residual stability is provided so that
the survival of the passengers/crew and ultimately the vessel and cargo.

On of the difficulties in doing this is the fact that damages are not planned (as distinct
from the design activities which work on planned state of affairs), and thus are
unpredictable. It is not known where the damage will occur and what the extent will be
– indeed it is unknown whether the vessel will sustain any damage during its lifetime at
all. This means that we have to consider the effects of possible damage scenarios and
their probability of occurrence. The probability of occurrence has been traditionally
incorporated into the damage stability regulations as a ‘factorial system’ in the practice
of watertight subdivision, but with the industry having taken a step towards ‘goal-
setting’ rather than ‘prescriptive’ regulatory regime, the so-called probabilistic
assessment of survivability of ships and passengers/crew is in vogue at present time.

Whatever the stability regulatory regime, we need to have some methods of dealing
with various damaged situations. Clearly we need to understand first, however, in what
way the ship is affected after a damage. We can consider here only the outwardly
visible effects which can be readily measured directly as follows:

Draught
The draught will change so that the displacement of the remaining unflooded part
of the ship is equal to the displacement of the ship before damage less the weight of
any liquids or other cargo which were in the space opened to the sea.

Trim
The underwater body shape changes and the flooded water also causes the centre of
gravity to change. It is obvious that a new state of equilibrium will involve an
alteration in trim.

Heel
If the flooding is asymmetrical, a static heel will be inevitable. A static heel poses a
danger of capsizal and, therefore, regulations demand that in such cases cross-
flooding arrangement be made so that port and starboard balance can be achieved
within a short time. Nevertheless, it is important to note that until such a balance
can be achieved, the ship may go through interim state of static heel.

Stability
It is reasonably obvious why the stability characteristics will alter, and this will be
discussed in more detail.

61
Freeboard
One of the unpleasant consequence of the increase in draught is the reduction in the
amount of freeboard available. Freeboard is a kind of reserve buoyancy and
stability, and its reduction can be serious. For example, the so-called ‘margin line’
is used to define the minimum freeboard allowed in any damaged states.

Loss of ship
Where changes to any one or more of the above factors are excessive beyond
certain limits, loss of ship can occur, either through capsizal, plunging, sinking,
structural breaking up or any combination of these processes.

It can be seen that the hydrostatics of a damaged ship is somewhat more complicated.
Essentially, however, it is still a floating body, and, as such, exactly the same principles
as used in intact hydrostatics can be applied. The subject of damaged stability and the
safety or survivability of damaged ships is a complex one and can easily form a
separate subject. The discussion in this part, therefore, is necessarily brief and only
deals with the most basic ideas.

5.2 INTERMEDIATE STAGES OF FLOODING

During the intermediate stages of flooding (i.e. from the time of damage to the time
when a final new equilibrium has been reached) water is continually flooding into the
damaged compartment(s). Flooding may not be ‘continuous’, as the water flow is
determined by the difference in head of the surface inside and out. On the outside the
surface is continuously changing due to wave action, while inside the water already
flooded in will ‘slosh’ about due to the vessel motion. Therefore, the water flow may
not even be one-way all of the time.

Nevertheless, short of tackling this with a time-domain simulation, some simplification


is necessary and usually the following assumptions are made:

- The vessel is assumed to be in static equilibrium at every stage of flooding, with


water surface in the flooded compartment parallel to (but at a lower level) the
surface of the sea.

- In the case of asymmetrical flooding involving spaces, which are cross-connected


by pipes, ducts, etc., it is normally assumed that the flooding water in the damaged
wing spaces reaches sea-level.

- During the intermediate stages of flooding, heel may occur either as a result of
negative residual GM or from asymmetrical flooding. Since we are dealing with a
transient state, some heel is acceptable provided it is not excessive and long-lasting
and also the range of stability and maximum righting arm include sufficient margin
of safety.

5.3 EQUILIBRIUM, DRAUGHT AND STABILITY AFTER FLOODING

In order to calculate the ship’s attitude in equilibrium, broadly speaking two methods
are used representing perhaps the two fundamental ways of looking at the flooded
water.

62
Lost Buoyancy Method

In this approach the damaged compartment is treated to be completely open to sea


and therefore no longer contributes to the ship’s buoyancy. Just imagine the
normally watertight hull being suddenly changed to sieves with large holes! The
compartments flooded are consequently removed completely from the hydrostatic
calculations. To compensate for this lost buoyancy, the ship has to sink further to
an increased draught until the displacement becomes identical to that of before the
damage, as the ship’s mass is assumed to be unchanged. Te centre of gravity of the
ship remains unchanged.

It is easy to see that this is a physical over simplification, but the method is simpler
to use than the added weight approach.

Added Weight Method

In this approach the damaged compartment is treated to be still intact, and the
flooded water is treated in exactly the same manner as ballast water taken on board.
In other words the mass displacement of the ship has been increased by the amount
of water ingress. An increase in draught occurs to provide the extra buoyancy for
this added weight of flood water. This of course will alter the centre of gravity of
the vessel as well.

The method is a little more complex than the LBM, but represents the reality a little
more closely. It is useful for dealing with the intermediate stages of flooding and
also when we wish to work out the length and location of the compartment which
can be flooded without the waterline going over the margin line (‘floodable length’
calculation).

It is important to note that both methods should produce identical values of the
physically measurable quantities. For example, draught and righting moment are some
of the items which can be measured, while the location of centre of gravity cannot be
measured as it is essentially a conceptual point. The main points of the difference
between the two approaches are summarised below:

LBM AWM
Displacement ∆ ∆ + δ∆
VCG ( KG )0 ( KG )1
Draught T + δT
VCB ( KB )1 ( KB )2
Righting moment R.M.
Righting arm R.M . ∆ R.M . ( ∆ + δ∆ )
Free surface effects n.a. yes

It is crucial, therefore, to understand that GM value alone is insufficient information


unless accompanied by a displacement used (or more commonly the method used to
calculate it, i.e. LBM or AWM).

63
Before we examine the two methods using some examples, we need to determine how
much water a damaged compartment can admit into it. This point is dealt with by a
factor called permeability, which is defined as the ratio of the volume which can be
occupied by flood water to the total nominal volume of the compartment. Typical
values of permeability are:

accommodation spaces 95%


machinery compartments 85%
coal bunkers, stores, cargo holds 60%
tanks 0 – 95%

Of course, cases can be made for differing values of permeability depending on


circumstances.

Example 1

To illustrate the general effects and principles, consider a box-shaped vessel as


being the only one in which the numerical work is not unduly laborious. Take a
box-shaped vessel, 120mL x 21mB x 8m even keel draught with KG = 6.0m. A
compartment of length 30m and spanning the whole breadth of the vessel, situated
amidships is flooded due to a damage to the hull. Assume 100% permeability.
Calculate the residual GM.

We will do this with both methods, but with LBM in the first instance.

LBM

original WPA = 120 x 21 = 2520m2


lost WPA = 30 x 21 = 630m2
Therefore, intact WPA = 2520 – 630 = 1890m2
intact TPC = 1890 x 1.025/100 = 19.37 tonnes/cm

lost buoyancy = 30 x 21 x 8 x 1.025 = 5166 tonnes


sinkage = 5166/1937 = 2.67m
new draught = 10.67m thus, KB = 5.33m

new BM =
(120 − 30 ) × 213 = 3.45m
12 × 120 × 21× 8

new KM = 5.33 + 3.45 = 8.78m


KG = 6.00m

Therefore, new GM = 8.78 – 6.00 = 2.78m

Remember, the displacement is still 120 x 21 x 8 = 20,664 tonnes.

AWM

Let new draught be T. Since original displacement plus the added weight is the
new displacement,
120 × 21× 8 ×1.025 + 30 × 21× T ×1.025 = 120 × 21× T ×1.025
Solving it for T we get
T = 10.67m and consequently KB = 5.33m

64
Now we must obtain the new KG

20, 664 × 6 + 6890 × 5.33


new KG = = 5.83m
( 20, 664 + 6890 )

30 × 213
free surface correction = = 3.45m
12 × 120 × 21× 10.67

Therefore residual GM = 5.33 + 3.45 – 5.83 – 0.86 = 2.09m

Remember the new displacement = 27,554 tonnes.

As can be seen from this example the residual GM values calculated by the two
methods are not the same. However, the righting moment which is directly
proportional to GM at small angles of inclination should be the same. Therefore we
should get identical values of (displacement x GM).

From the LBM 20,664 x 2.78 = 57,500


From the AWM 27,554 x 2.09 = 57,500.

When the flooding is not symmetrical, the change in GM is accompanied by a heeling


moment. The resulting heel makes the added weight method quite clumsy to use as will
be illustrated by a simple case here. Assume a compartment is flooded as shown. With
the LBM, it is quite easy to calculate the lost buoyancy, and thus the sinkage to W1L1.
Then, of course due to the imbalance in buoyancy a static heel occurs which will be
about the new transverse centre of flotation, F. With the AWM, however, we cannot
determine W1L1 directly because we do not know the amount of flooded water until that
waterline is determined. Even when that is done, as indeed it can be done through a
graphic method, problem does not go away, because the heeling alters added weight yet
again and this in turn changes the heeling moment and so on. This time, it is more
difficult to decide the heel angle. To overcome this problem a hybrid method, called
the quasi-added weight method, is sometimes used.

Example 2

Consider the previous box-shaped vessel, 120mL x 21mB x 8m even keel draught
with KG = 6.0m. A compartment of length 30m and spanning the entire starboard
half-breadth of the vessel, situated amidships is flooded due to a damage to the hull.
Although it is a quite meaningless exercise, we wish to find the original KG of the
vessel before flooding when the resulting angle of heel is such that
tan φ = 0.25 or φ ≈ 14° . This simplifies the problem so that we can examine it
here. Assume 100% permeability.

LBM

original TPC = 25.83


lost TPC = 3.23

intact TPC = 22.60 tonnes/cm

lost buoyancy = 1.025 x 30 x 10.5 x 8 = 2583 tonnes

65
parallel sinkage = 1.14m
new draught = 9.14m KB = 4.57m

For finding new BM

Area lever from 1st mt 2nd mt IT own CG IT


CL
original 2520 0 0 0 92610 92610
WP
lost WP -315 5.25 -1653.75 8682 2894 -11574
Residual 2205 1653.75 81034

CF = 1653.75/2205 = 0.75m from the centreline.

Therefore the residual IT about the new TCF is

IT = 81,034 – 2205 * 0.752 = 79,794m4

Displacement = 20,160m3
BM = 79794/20160 = 3.96m

In this particular example, since the compartment is flooded to the bottom shell
and the vessel is of box-like shape, the transverse shift of the centre of
buoyancy is the same as the transverse shift of the centre of flotation, viz.
0.75m. The centre of gravity of the vessel is still on the centreline, and,
therefore, the heeling moment is ∆ × 0.75 cos φ . Using the wall-sided formula
and equating the heeling moment to the righting moment,

3.96
0.75 = GM tan14° + tan 3 14°
2
giving GM = 2.88m

since KM = 8.53m, KG = 5.65m.

AWM

Let the draught on the centreline after flooding be T.


Original displacement + added weight = new displacement
Thus,
120 × 21× 8 ×1.025 + 30 ×10.5 (T + 5.25 tan14° ) ×1.025 = 120 × 21× T ×1.025
Therefore, T = 9.33m.

KB = 4.66m and new displacement = 24,108 tonnes.


added weight = 3444 tonnes.

CG of the flooded water


⎛ a + 2b ⎞ h 9.33 + 2 × 11.96 10.5
=⎜ ⎟ = × = 5.47 m from the C.L.
⎝ a+b ⎠ 3 9.33 + 11.96 3

heeling lever of the flooded water = 3444 x 5.47 / 24,108 = 0.781m

BM = (21 x 21)/(12 x 9.33) = 3.94m

66
Using wall-sided formula,

3.94
0.781 = GM tan14° + tan 3 14°
2
resulting in GM = 3.00m

KM = 8.60m

Therefore, KG = 5.60m in flooded condition.

The v.c.g. of the flooded water is the vertical position of the trapezoidal shape
and it is 5.35m.

Tonnes v.c.g. Moment


24108 5.60 135,005
-3444 5.35 -18425
_____________________________________
20,664 116,580

Therefore original KG = 5.65m.

Please note that no free surface correction was necessary, as we already had the
waterline at the final equilibrium and that the true centre of gravity of the flood
water was used for calculating the heeling moment. The alternative was to take
the CG of the flood water at the centre of the tank and make a free surface
correction in the righting moment, or

moment of added water = 3444 x 5.25 = 18,081


heeling lever = 0.750m
giving GM = 2.88m
f.s.e. = 2894/2418 = 0.12m
corrected GM = 3.00m.

In the above two examples the flooding was limited to midship compartments or at
worst symmetrical flooding about midship was assumed. In general flooding due to
damage, however, is more likeley to be asymmetrical longitudinally. We need,
therefore, to include the effects of flooding on trim. This is nearly always done with the
LBM and the process is roughly summarised as follows:

(a) calculate permeable volume of compartment up to original WL;


(b) calculate TPC, long’l and transverse positions of CF with the damaged area
removed;
(c) calculate revised second moments of waterplane area about the CF in the two
directions and, hence new BMT and BML.
(d) calculate parallel sinkage and the rise of CB due to vertical transfer of buoyancy
from the flooded compartment to the parallel layer;
(e) calculate new GM’s
(f) calculate angles of rotation due to the eccentricity of the loss of buoyancy from the
new CFs.

Example 3

67
A compartment having a plan area at the waterline of 100m2 and centoid 70m fwd
of midships, 13m to starboard is bilged. Up to the waterline obtaining before
bilging, the compartment volume was 1000m3 with the centres of volume 68.5m
fwd of midships, 12m to starboard and 5m above keel. The permeability was 0.70.

Before the incident the ship was floating on an even keel draught of 10m at which
the following particulars are given

∆ = 30, 000 tonnes KML = 170m


KG = 9.40m WPA = 4540m2
KMT = 11.40m LCF = 1m fwd of midships
KB = 5.25m LBP = 220m

Calculate the heel and trim when the compartment is bilged.

Use lost buoyancy method.

permeable volume = 0.7 x 1000 = 700m3

damaged WPA = 4440m2


−100 × ( 70 − 1)
movement LCF = = −1.55 , i.e. LCF moves by 1.55m aft.
4440
new LCF = -0.55m, or 0.55m aft of midships.

−100 × 13
movement TCF = = −0.29m or 0.29m to port.
4440
new TCF = -0.29m.

original IT = (11.40 − 5.25 ) × 0.975 × 30, 000 = 179.889m


4

damaged IT = 179,889 − 100 × 132 − 4440 × ( 0.29 ) = 162, 620m 4


2

(we have ignored I of the damaged part about own axis for speed)

162, 620
damaged BM T = = 5.56m
0.975 × 30, 000

original I L = (170.0 − 5.25 ) × 0.975 × 30, 000 = 4.819 ×10 m


6 4

damaged I L = 4.819 × 106 − 100 ( 69 ) − 4440 (1.55 ) = 4.332 × 106 m 4


2 2

4.332 ×106
damaged BM L = = 148.1m
0.975 × 30, 000

parallel sinkage = 700/4440 = 0.16m

700 (10 + 0.08 − 5 )


rise of CB = = 0.12m
0.975 × 30, 000

damaged GM T = 5.25 + 0.12 + 5.56 − 9.40 = 1.53m


damaged GM L = 5.25 + 0.12 + 148.1 − 9.4 = 144.1m

68
700 ×12.29 180
Angle of heel = × = 11.0°
0.975 × 30, 000 ×1.53 π

700 × ( 68.5 − 1 + 1.55 )


Angle of trim = 0.01147radians
0.975 × 30, 000 ×1.53

Change of trim = 0.01147 × 220 = 2.52m between perpendiculars.

69

Potrebbero piacerti anche